Practice Set Questions

Lakukan tugas rumah & ujian kamu dengan baik sekarang menggunakan Quizwiz!

A solo practitioner limits her law practice to two kinds of cases: medical malpractice and products liability. When the solo practitioner turned 67, she began looking for someone to buy her law practice. Ultimately, she sold the medical malpractice part of her practice to lawyer Alpha for $400,000, and she sold the products liability part of her practice to attorney Beta for $250,000. The solo practitioner then retired. Within 30 days after the sales to Alpha and Beta, approximately 40% of the solo practitioner's former clients decided to collect their case files and take their business to different lawyers. Were the sale from the solo practitioner to Alpha and the sale from the solo practitioner to Beta proper? A Yes, even though the solo practitioner sold pieces of her practice to two different buyers, and even though 40% of the solo practitioner's clients left the buyers within 30 days. B Yes, but the solo practitioner is subject to civil liability to the two buyers for unjust enrichment because 40% of the solo practitioner's clients left the buyers within 30 days. C No, because the solo practitioner sold pieces of her practice to two different buyers. D No, because 60% of the solo practitioner's clients stayed with the buyers for more than 30 days.

A. ABA Model Rule 1.17 permits a lawyer to sell her entire law practice, or an area of her law practice, to one or more lawyers or law firms. Here, the solo practitioner sold her entire law practice to two different lawyers, and ABA Model Rule 1.17 permits that. The departure of 40% of the solo practitioner's clients does not cause the sales to violate ABA Model Rule 1.17. Indeed ABA Model Rule 1.17(c)(2) requires the selling lawyer to notify her clients that they have a right to pick up their files from the buyer and take them to a different lawyer. [See also comment 2 to ABA Model Rule 1.17] (B) and (D) are wrong because clients are not like sheep that can be bought and sold, willy-nilly. To hold the solo practitioner either subject to discipline or civilly liable in unjust enrichment would be inconsistent with the clients' right to pick up their files and take them to a different lawyer. The solo practitioner, Alpha, and Beta made their sales contracts in the context of ABA Model Rule 1.17 so Alpha and Beta cannot claim that they were surprised when clients departed. (C) is wrong because ABA Model Rule 1.17(b) permits a lawyer to sell her entire practice to one or more lawyers or law firms. One might also argue that the solo practitioner's medical malpractice cases are in a different "area of practice" (professional malpractice) from her products liability cases. But that argument is not necessary here because the solo practitioner sold her entire law practice.

A paralegal works for the law firm of Alpha & Beta. Her direct supervisor is partner Alpha, whose practice is limited to international trade law. Partner Beta is the firm's leading trial lawyer, both in commercial and personal injury cases. On her way to work one morning, the paralegal saw a pedestrian run down in a crosswalk by a speeding car. The paralegal rendered first aid, and while she was waiting with the pedestrian for the ambulance, the paralegal provided the pedestrian with a business card and urged him to call the firm to obtain legal representation in connection with his injuries. When she got to work, she told partner Alpha what she had done. Alpha admonished the paralegal not to hand out the firm's cards in such situations, but he did not discuss the matter with partner Beta. Is Alpha subject to discipline? A Yes, because he failed to warn Beta not to take the pedestrian's case. B Yes, because as the paralegal's supervisor, he is responsible for any unethical act she commits. C No, because as a nonlawyer, the paralegal is free to recommend a lawyer to someone if she wishes. D No, because the paralegal may not have been aware at the time that she did anything wrong.

Alpha is subject to discipline for failing to warn Beta not to take the case. If the paralegal were a lawyer, her conduct would violate ABA Model Rule 7.3(a), which prohibits in-person solicitation. The partners in a firm are responsible for educating their nonlawyer employees about ethics issues and making reasonable efforts to assure that those employees comply with ethics rules. [ABA Model Rule 5.3(a)] Moreover, a partner is subject to discipline if he learns about the violation of an ethics rule by a nonlawyer employee "when its consequences can be avoided or mitigated," but the partner "fails to take reasonable remedial action." In this case, the consequences of the paralegal's solicitation could have been avoided by warning Beta not to take the pedestrian's case. Because he failed to warn Beta, Alpha is subject to discipline. (B) is wrong because it is too broad. A lawyer's responsibility for a nonlawyer employee's ethics violation is limited to situations in which the lawyer orders it, ratifies it, or learns about it in time to remedy it and does not do so. [ABA Model Rule 5.3(c)] (C) is wrong because even though people are generally free to recommend a lawyer to someone else, that does not allow the paralegal to solicit business for the firm that employs her. [ABA Model Rule 8.4(a)] (D) is wrong because Alpha and the other partners in the firm had a duty to educate the paralegal about ethics rules. [ABA Model Rule 5.3(a)] Furthermore, even if the paralegal acted innocently, that does not excuse Alpha's failure to warn Beta not to take the case.

An attorney is a voting member of the legislation committee of a consumer-based law reform group that drafts and advocates the passage of proposed statutes on food safety. The law reform group is currently debating a draft statute that sets quality and safety standards for growth hormones administered to chickens, turkeys, and other poultry. The attorney is also engaged in the private practice of patent law. She regularly represents a biotechnology firm. Using the techniques of genetic engineering, the biotechnology firm invents, develops, and sells a variety of patented growth hormones. The attorney herself has obtained patents on some of these hormones for the biotechnology firm. If enacted into law, the law reform group's proposed statute on poultry hormones could materially increase the biotechnology firm's hormone sales because it is the only firm whose hormones would meet the statute's quality and safety requirements. Would it be proper for the attorney, as a member of the law reform group's legislation committee, to participate in the debate on, and to cast her vote on, the proposed statute? response - incorrect A No, because the statute could materially benefit the biotechnology firm. B No, because the attorney may not serve as a member of the law reform group while representing the biotechnology firm. C Yes, provided that she informs the legislation committee that she represents an unnamed client whose interests could be materially benefited by the statute. D Yes, provided that she informs the legislation committee that she represents the biotechnology firm, whose interests could be materially benefited by the statute.

C. must say she represents someone but doesn't need to say who t would be proper for the attorney to participate in the debate and cast her vote on the proposed legislation, provided that she informs the committee that she represents a client whose interests could be materially benefited by the statute. A lawyer may participate in a law reform activity that will affect the interests of the lawyer's client. [ABA Model Rule 6.4] When a lawyer knows that a client will be materially benefited by the activity, the lawyer must disclose that fact, but she need not name the client. (A) is wrong because a lawyer is not prohibited from engaging in a law reform activity that might benefit her client. (B) is wrong because a lawyer is not prohibited from participating in a law reform activity, unless the participation would create an impermissible conflict of interest. [ABA Model Rule 1.7(a)] That is not the case here. A client who hires a lawyer does not thereby purchase the right to control the lawyer's views and activities in all contexts. [See ABA Model Rule 1.2(b)] The attorney may even advocate new legislation that she thinks is sound that would harm the biotechnology firm's sales. [See ABA Model Rule 6.4] (D) is wrong because the attorney need not disclose the name of her client; simply disclosing the fact of representation will inform the legislation committee of her possible bias.

A lawyer is one of only nine lawyers who practice probate law in a particular county. In that county, all probate matters go before a single judge—the probate judge. The probate judge's duties include appointing counsel for the administrators of intestate estates. Serving as an administrator's counsel can be very lucrative. The incumbent probate judge recently retired. Her custom was to appoint out-of-county lawyers to serve as administrators' counsel, believing that such lawyers are less subject to local political and social pressures than county lawyers. The probate judgeship will be filled in six months in a partisan, contested election, and the lawyer is one of the candidates. The lawyer met jointly with the county's eight other probate lawyers and said he was seeking the probate judgeship, and would be making some important changes, such as appointing only local lawyers as counsel for administrators of intestate estates. He also said that if they'd like to contribute money or time, to please get in touch with his campaign committee via his website. Five of the eight lawyers sent generous monetary donations to the lawyer's campaign committee for the self-confessed purpose of securing legal appointments if the lawyer wins. The other three lawyers volunteered generous amounts of their time in the lawyer's campaign, but their motives for doing so were unexpressed and unclear. Which of the following is correct? A The lawyer is subject to discipline for personally soliciting money and publicly stated support from the eight lawyers. The five lawyers who contributed money will be subject to discipline if the lawyer wins and if they accept appointments from him. B The lawyer's conduct was proper because he was a candidate for a judicial office in a contested election. All eight lawyers, however, are subject to discipline because a practicing lawyer must not contribute either money or time to the political campaign of a judge before whom the lawyer expects to appear. C The lawyer is subject to discipline for promising to appoint only local lawyers as counsel for administrators, in the hope of securing the publicly stated support of the eight lawyers. The conduct of all eight lawyers, however, was proper because lawyers are permitted to support or oppose candidates in contested elections for judgeships. D The lawyer's conduct was proper because he was a lawyer-candidate, not a judge, at the time he met with the eight lawyers. The conduct of the five lawyers who contributed money was proper because a person's motive for making a political contribution is a private matter that is protected by the First and Fourteenth Amendments.

The lawyer is subject to discipline under CJC Rule 4.1(A)(8), which prohibits a judicial candidate from personally soliciting campaign contributions or publicly stated support. The five lawyers who contributed money will be subject to discipline if the lawyer wins and if they accept appointments from him. [See ABA Model Rule 7.6] (B) is wrong because the lawyer is subject to discipline under CJC Rule 4.1(A)(8), as noted above. The second sentence of (B) is wrong because it overstates the constraints on a lawyer's participation in a judicial campaign. (C) is wrong because the five lawyers who contributed money violated ABA Model Rule 7.6, as noted above. The first sentence of (C) is a misapplication of CJC Rule 4.1(A)(13), which prohibits a judge from making pledges or promises that are inconsistent with the impartial performance of the judge's adjudicative duties "in connection with cases, controversies, or issues that are likely to come before the court." Here, the policy regarding whom the probate judge will appoint as administrators' counsel is not the kind of litigation issue to which the highlighted language of CJC Rule 4.1(A)(13), above, refers. The second sentence of (C) is wrong because the motive of the five money contributors was to obtain appointments if the lawyer won. [See ABA Model Rule 7.6] (D) is wrong because a lawyer who runs for a judicial post must follow the CJC. [See CJC Rule 4.1, comment 2] The second sentence of (D) is wrong because ABA Model Rule 7.6 trumps whatever privacy rights one can find in the peripheral glow of the First and Fourteenth Amendments.

Two sisters are partners in a bakery. Their partnership agreement says that they will share the work and the profits equally. They are very close, but they constantly bicker—each claims that the other is taking an unfair share of the profits and shirking on the work. Six months ago, they hired a lawyer to act as a third-party neutral, to help them resolve their differences once and for all. At the outset, the lawyer explained that he would be strictly neutral between them; he would not be representing either one, and neither of them would be entitled to the protections afforded by an attorney-client relationship. After a long series of meetings with them (sometimes separately, sometimes jointly), the lawyer proposed a solution. The sisters liked his solution, reduced it to writing, and signed it, vowing to end their bickering forever. Six months later, the feud erupted again, worse than ever. One of the sisters asked the lawyer's law firm to represent her in a lawsuit against her partner-sister, seeking to declare the partnership at an end and to bar her partner-sister from entering the bakery premises. Which of the following is correct? response - incorrect A The lawyer is subject to discipline for his failed effort to serve both sisters when their interests were patently in conflict. B It would be proper for the lawyer to represent the sister in the lawsuit as she requested, even without the informed consent of her partner-sister. C The lawyer's law firm partner may represent the sister in the lawsuit as she requested, but only if her partner-sister is notified in writing, and only if the lawyer is timely screened and does not share in the fee earned in the lawsuit. D The lawyer's law firm partner would be subject to discipline for representing the sister in the lawsuit as she requested, even if the lawyer is timely screened and does not share in the fee earned in the lawsuit.

c. The lawyer was not representing either sister; rather, he was acting as a third-party neutral to help them resolve their differences. ABA Model Rule 2.4 permits a lawyer to serve in that role. ABA Model Rule 1.12(c) permits screening to avoid a conflict in this situation. Therefore, (A) and (D) are incorrect. (B) is incorrect because, when a lawyer has served as a third-party neutral between two conflicted parties, he cannot later represent one of the parties in that matter, unless both parties give informed consent, confirmed in writing. [ABA Model Rule 1.12(a)]

Before a judge was elected to the bench, she and her law partner purchased a piece of property to be held in co-tenancy by the judge and her law partner. After the judge was elected to the bench, she agreed to pay her law partner an annual fee to manage the property because her own time would be severely limited by her judicial duties. The judge and her law partner meet every three months to discuss the status of the property. The law partner sometimes appears as an attorney in the judge's courtroom. Was it proper for the judge to make this arrangement with her law partner? A Yes, because the judge acquired the property before she became a judge. B Yes, because the judge made arrangements to ensure that her judicial duties would not suffer. C No, because judges should not engage in remunerative outside enterprises. D No, because the law partner appears in cases before the judge's court.

A judge should refrain from financial and business dealings that involve her in frequent transactions with lawyers or persons likely to come before the court on which she sits. [CJC Rule 3.11(C)(3)] Here, the judge's close business relationship with a lawyer who appears in her courtroom violates CJC Rule 3.11(C)(3). It makes no difference that the judge acquired the property before she became a judge. Thus, (A) is incorrect. (B) is incorrect because even if the judge's dealings with the property do not take up too much of her time, there will still be an adverse reflection on her impartiality. (C) is incorrect because CJC Rule 3.11(A) permits a judge to hold investments, including real estate, and to engage in other remunerative activity.

An attorney knows that the statute of limitations on her client's claim has run. However, the statute of limitations is an affirmative defense that the defendant in the case would waive if she failed to plead it. The attorney's client is willing to incur the legal fees and court costs of filing the lawsuit, and understands the risks. What may the attorney do? A File the suit because her client is willing to incur the legal fees and court costs. B File the suit but inform the court that the statute of limitations has run. C Not file the suit unless her client consents to disclose to the court the fact that the statute of limitations has run. D Not file the suit as it is now a frivolous claim.

A. (D) is incorrect; the claim is not frivolous under ABA Model Rule 3.1. The statute of limitations merely destroys the remedy and not the right. Unless the defendant pleads the statute of limitations, the claim is valid. Thus, (A) is correct. (B) and (C) are incorrect; there is no duty to inform the court that the statute of limitations has run.

An assistant district attorney ("ADA") has been assigned to prosecute a vagrant for petty theft and attempted sale of stolen property. The vagrant's arrest was the result of information provided by a local pawnbroker. The pawnbroker has himself been in trouble with the law on prior occasions, but he is not suspected of any present crime. The vagrant is represented by a public defender. The ADA wants to interview the pawnbroker for possible use as a prosecution witness. Which of the following is correct? response - incorrect A The ADA may interview the pawnbroker without the public defender's consent. B The ADA may interview the pawnbroker without the public defender's consent, but he would be subject to discipline for inquiring about the pawnbroker's prior criminal record. C The ADA would be subject to discipline if he interviewed the pawnbroker without the public defender's consent. D The ADA may interview the pawnbroker, but only with the public defender's consent, and the ADA would be subject to discipline for inquiring about the pawnbroker's prior criminal record.

A. A lawyer does not need the consent of adversary counsel to interview a nonparty witness. Furthermore, nothing in the Rules would prevent the ADA from asking the pawnbroker about his prior criminal record, and he would have good reason for doing so because the pawnbroker's criminal record can be used to impeach his testimony. Thus, (A) is correct, and (B), (C), and (D) are incorrect

A married couple experienced continuing marital difficulties and, being unable to resolve their problems, decided to divorce. The wife retained an attorney, who filed papers on the husband. Shortly after he was served, the husband telephoned his wife's attorney and suggested that the matter could be resolved amicably. The wife's attorney asked the husband if he was represented by counsel. The husband replied that he was not. The wife's attorney strongly urged the husband to retain a lawyer to represent him and to safeguard his interests, but the husband insisted that he did not need a lawyer. A few weeks later, after ascertaining that the husband still had not retained counsel, the wife's attorney sent the husband a proposed settlement dividing the marital property. There were spaces at the bottom for both the husband and the wife to sign, but the copy sent to the husband was unsigned by the wife. In his cover letter to the husband, the wife's attorney indicated that the husband could sign the settlement agreement, which the attorney characterized as being fair and equitable. However, the attorney's letter also stated that he could not advise the husband as to whether he should sign, and strongly urged the husband to obtain independent counsel to review the papers before signing them. Were the attorney's actions in handling the divorce proper? A Yes, because the attorney did not give the husband legal advice. B Yes, because the attorney urged the husband to obtain representation. C No, because the attorney did not have the wife sign the settlement papers before sending them to the husband. D No, because the attorney should have required that the husband have the papers reviewed by an attorney.

A. ABA Model Rule 4.3 forbids an attorney to give advice to an unrepresented person if the lawyer knows that the person's interests conflict with those of the client. Thus, (A) is correct. The attorney is allowed to advise the unrepresented person to obtain a lawyer, but the critical issue is giving advice. Thus, (A) is a better answer than (B). (C) is irrelevant to any ethical issue. (D) is incorrect because there is no way a lawyer can compel a third party to obtain counsel.

A judge serves on a state trial court that has nine other judges. Her husband is a life insurance salesman for a large life insurance company. The life insurance company is occasionally a litigant in the court on which the judge sits. Every year the life insurance company runs a national sales contest in which the person who sells the most life insurance during the year receives a valuable prize. The judge's husband won this year and took the judge on an all-expense-paid vacation in Europe. She did not make a public report of the prize. Was it proper for the judge to allow her husband to accept the prize and take her on the European vacation? A Yes, because acceptance of the prize cannot reasonably be perceived as undermining the judge's integrity or impartiality. B Yes, because the prize was won by her husband, not by the judge. C No, because the judge did not make a public report of the prize. D No, because the life insurance company may later appear as a litigant in the court on which the judge sits.

A. It was proper for the judge to allow her husband to accept the prize because acceptance thereof does not reasonably appear to undermine the judge's integrity or impartiality. A judge may accept benefits associated with her spouse's business activity that incidentally benefit the judge. [CJC Rule 3.13(B)(8)] (B) is wrong because it ignores the general rule on family members' accepting gifts and other benefits. (C) is wrong because such benefits of a spouse are not subject to the public reporting requirement. [CJC Rule 3.13(B)] (D) is wrong because if the prize is proper under CJC Rule 3.13(B)(8), it does not become improper simply because the insurance company may later appear as a litigant in the judge's court.

An insurance company offers a legal services insurance policy. In return for a yearly premium, an insured will be reimbursed by the insurance company for a specified amount for legal services during the year. The insured selects a lawyer from a list of "authorized providers" supplied by the insurance company. Any lawyer who agrees to follow a maximum fee schedule set by the insurance company can become an "authorized provider." The insurance company solicits insurance sales by in-person and live telephone contact with potential insurance buyers, working systematically through local telephone directories. Will an attorney be subject to discipline if he becomes an "authorized provider" and receives clients through the insurance company's insurance plan? A No, because the insurance company does not specifically target persons whom it knows are in need of legal services in a particular matter covered by its insurance plan. B No, because the insurance company's insureds are allowed to select whatever lawyer they wish from among the "authorized providers." C Yes, because the insurance company uses a specified maximum fee schedule. D Yes, because the insurance company uses in-person and live telephone solicitation to get business.

A. The attorney will not be subject to discipline if he becomes an authorized provider under the insurance company's plan. The insurance company has set up a prepaid legal services plan of the kind referred to in the ABA Model Rules. A lawyer may receive legal business through such a plan, unless the operator of the plan uses in-person or live telephone contact to solicit people who it knows are in need of legal services in a particular matter covered by the plan.

A lawyer is a partner in a private law firm. That firm regularly provides legal services to three major banks and two other important lending institutions in the community. The lawyer has been invited to become a member of the board of directors of the local legal aid society, the group that sets overall governing policies for the local legal aid office. One of the major issues that will soon face the board of directors is whether to amend the case intake guidelines to allow the legal aid office to represent clients in disputes with banks and other lending agencies. Which of the following statements is correct? A The lawyer may join the board of directors, but she must refrain from participating in the decision about the case intake guidelines. B The lawyer will be subject to discipline if she joins the board of directors because service on the board is in conflict with the interests of her firm's bank and lending institution clients. C It would be proper for the lawyer to join the board of directors, and it would be proper for her to participate in the decision about the case intake guidelines. D The lawyer may join the board of directors to help discharge her pro bono obligation, and she may vote in favor of amending the case intake guidelines in order to make it easier for low income persons to sue banks and other lending institutions.

A. The lawyer may join the board of directors, but she must refrain from participating in the decision about the case intake guidelines. A lawyer may not participate in a legal service board decision that may adversely affect one of the lawyer's clients. [ABA Model Rule 6.3(b)] (B) is wrong because ABA Model Rule 6.3 encourages work with a legal services organization, even if the organization serves people whose interests conflict with the interests of the lawyer's clients. (C) and (D) are wrong because ABA Model Rule 6.3(b) prohibits a lawyer from taking part in a legal services organization decision if the decision will adversely affect one of the lawyer's clients.

An attorney limits her law practice to the representation of plaintiffs in actions for medical malpractice. She has developed a standard employment contract to use with all clients who desire a contingent fee arrangement. The contract requires the client to pay in advance the first $500 of litigation expenses (which is a reasonable sum in the ordinary medical malpractice case), and states that the attorney's fee shall be 20% of any amount recovered without going to trial, 30% of any amount recovered if the case goes to trial, and 35% of any amount recovered if the case is appealed. A client signed the attorney's standard form contract, and after putting in only 10 hours' work on the client's case, the attorney was able to work out a settlement agreement in which the client received $10,000 from the defendant's malpractice insurance carrier. The client was delighted with the settlement, but he is unhappy with having to pay 20% of it ($2,000) to the attorney for so few hours' work. With respect to the clause of the attorney's standard form contract that requires the client to pay in advance the first $500 of litigation expenses, which of the following is correct? response - incorrect A The clause is proper. B The attorney must delete the clause because it is an effort to impose a uniform provision on all clients, irrespective of their particular needs and situations. C The attorney is subject to discipline for using the clause because an attorney is required to advance reasonable litigation expenses to contingent fee clients. D The attorney is subject to discipline for using the clause because clients are required to pay litigation expenses as they are incurred.

A. There is nothing in the ABA Model Rules that prohibits a lawyer from requiring a client to pay in advance the first $500 of litigation expenses, so long as the $500 payment is reasonable. [ABA Model Rule 1.5(a)] (B) is incorrect because the ABA Model Rules do not suggest that every clause of a standard form employment contract must be individually tailored to each client. (C) is incorrect because ABA Model Rule 1.8(e) permits lawyers to advance litigation expenses, but does not require them to do so. (D) is incorrect because nothing in the Rules requires the client to pay litigation expenses as they are incurred.

A full-time judge lives in State A. Her father lives in a retirement home in State B. The judge's father told her that several of his friends in the retirement home had employed an attorney to write wills for them, and that in each will the attorney had included a bequest to himself. Each bequest was approximately 50% of the estimated total value of the person's probable estate. The friends told the judge's father that they did not really want to leave the attorney anything, but they had assumed it was merely a matter of routine, a part of the attorney's compensation for drafting the will. The attorney is admitted to practice in State B, but not in State A. The judge did not talk personally with any of her father's friends, but she believes that her father's rendition of the story is entirely accurate. Would it be proper for the judge to communicate directly with the attorney about the matter, and if that does not satisfy her, to communicate with the attorney disciplinary authority in State B about the matter? response - correct A Yes, because she has received information indicating a substantial likelihood that the attorney has violated a legal ethics rule. B Yes, because she has personal knowledge that the attorney has violated a legal ethics rule. C No, because legal ethics violations that take place outside State A are not her concern. D No, because she is not allowed to communicate directly with the attorney about the supposed legal ethics violation.

A. Yes, it would be proper for the judge to take the steps mentioned because she has received information indicating a substantial likelihood that the attorney has violated a legal ethics rule. The attorney has apparently violated ABA Model Rule 1.8(c), which generally prohibits a lawyer from drafting a will under which he will receive a substantial gift. A judge who receives "information indicating a substantial likelihood" that a lawyer has violated a legal ethics rule must take "appropriate action," which may include direct communication with the lawyer, direct action, and reporting the violation to the appropriate authority. [CJC Rule 2.15(D)] (B) is wrong because it is factually incorrect—the judge does not have personal knowledge; rather she has received second-hand information about the attorney. [See CJC, Terminology] (C) is wrong because the duties imposed by CJC Rule 2.15 are not confined to lawyers in the judge's own jurisdiction. (D) is wrong because "appropriate action" may include direct communication with the lawyer who violated the legal ethics rule. [CJC Rule 2.15(D), comment 2]

An attorney represented a defendant in a criminal trial. After the jury returned a guilty verdict, the defendant was taken to jail and the jury was discharged. While walking to his car, the disappointed attorney spotted one of the courtroom spectators in the parking lot. The attorney recalled that the spectator had been a member of the jury pool, but he had exercised a peremptory challenge against her because he instinctively felt that she would vote against the defendant. Despite not being selected as a juror, the spectator developed an interest in the case and had attended the entire trial. In an attempt to determine whether his instinct during jury selection was correct, the attorney approached the spectator and asked her whether she would have voted to convict the defendant. The spectator said, "I'd rather not talk about it." When the attorney explained that he was simply looking for constructive feedback, the spectator changed her mind and agreed to a brief interview. The attorney and spectator spoke for a few minutes, and the communication did not involve misrepresentation, coercion, duress, or harassment. Is the attorney subject to discipline? A Yes, because the spectator initially declined to speak with the attorney. B Yes, because post-trial contact with prospective jurors is prohibited. C No, because the communication did not involve misrepresentation, coercion, duress, or harassment. D No, because the spectator was not chosen for the jury.

A. The attorney is subject to discipline because the spectator initially declined the attorney's request for an interview. ABA Model Rule 3.5(c) provides that after the trial is over and the jury is discharged, a lawyer must not communicate with a former juror or prospective juror if any of the following conditions is met: (i) local law or a court order prohibits such communication; (ii) the juror has told the lawyer that she does not want to communicate; or (iii) the communication involves misrepresentation, coercion, duress, or harassment. Here, the attorney violated the second condition—he persisted with his interview request after the spectator said that she did not want to talk with him. (C) is incorrect. Even though the communication did not involve coercion, duress, or harassment, the attorney still spoke with the spectator after she declined his request, violating the rule. (D) is incorrect because ABA Model Rule 3.5(c) applies to all jurors and even prospective jurors. (B) is too broad. There is no blanket prohibition regarding post-trial contact with jurors and prospective jurors. Rather, such communications are subject to conditions, and the attorney violated one of these conditions.

An attorney received her law degree two years ago from a small local college of law and technical sciences. Last summer she attended a three-day trial practice seminar at the Harvard Law School. During her brief career, she has tried five cases—two jury trials and three bench trials. She won both of the jury trials and two of the three bench trials. The attorney placed an ad under the subject heading "Trial Lawyers" in the classified pages of the local phone book. Her ad states in relevant part: "Trial Attorney Harvard Trained Never Lost a Jury Trial" Which of the following is correct? A To make the ad proper, the references to "Harvard Trained" and "Never Lost a Jury Trial" must be deleted. B To make the ad proper, the references to "Trial Attorney" and "Harvard Trained" must be deleted. C To make the ad proper, the references to "Trial Attorney" and "Never Lost a Jury Trial" must be deleted. D The ad is proper as written.

A. The attorney would be subject to discipline for the last two statements in her advertisement. It is misleading for her to state that she is "Harvard Trained," because reasonable readers could interpret that to mean that she received her law degree from that school. [See ABA Model Rule 7.1 and comment 2] The statement "Never Lost a Jury Trial," although literally true, could create unjustified expectations and is therefore misleading. [See ABA Model Rule 7.1 and comment 3] The reference to "Trial Attorney" would not make the lawyer subject to discipline. Given her brief time in law practice, she has had significant experience as a trial lawyer. A lawyer is allowed to state the fields of law in which she does or does not practice. [ABA Model Rule 7.4(a)] (B), (C), and (D) are all incorrect because they would permit "Harvard Trained" or "Never Lost a Jury Trial" (or both) to remain in the advertisement.

After graduating from law school, an attorney was admitted to practice in one state and not in any other jurisdiction. She joined the United States Army Judge Advocate General's ("JAG") Corps—the corps of lawyer-soldiers who provide legal services to the Army throughout the world. After completing her officer training and her training in military law, she was assigned to the JAG office at a military base in a different state. Even though she was not admitted to practice in that state, she was assigned to the legal assistance desk. According to Army regulations, her job is to provide legal services to military personnel and their dependents concerning a wide range of personal legal problems, including civil, domestic, and financial matters. An officer and his wife ask the attorney for legal advice about financing a mobile home, which they plan to put in a mobile home park located in the town closest to the military base. The attorney knows absolutely nothing about the business and legal issues involved in financing a mobile home, but she is willing to undertake additional research to learn about these issues. Would it be proper for the attorney to give the requested advice to the officer and his wife? A Yes, because she is willing to do the research necessary to give competent advice on mobile home financing. B No, because she is not knowledgeable about these business and legal issues. C No, because she is not admitted to practice general civil law in the new state. D No, because mobile home financing is not directly related to the Army's mission.

A. The attorney does not need to be admitted to practice in the new state because the JAG Corps is an organ of the federal government, and Army regulations authorize JAG officers to provide legal services to Army personnel and their dependents on a wide range of personal legal problems. [See ABA Model Rule 5.5(d)(2)] But before she advises the officer and his wife, the attorney must do enough research to become competent on the legal aspects of mobile home financing. [See comment 2 to ABA Model Rule 1.1] (B) is wrong because she is willing to undertake the additional study necessary to become competent in these matters, and therefore her current lack of knowledge is not dispositive. [See comment 2 to ABA Model Rule 1.1] (C) is wrong for the reason stated in the explanation of (A), above. (D) is wrong because the problem specifies that Army regulations authorize a legal assistance officer to give legal advice on a wide range of personal legal problems that affect military personnel and their dependents. These regulations reflect the Army's strong interest in keeping its people out of legal troubles no matter what the source.

After a major airplane crash in the vicinity of an affluent island town, in which 122 passengers and crew were killed, the town's attorneys swarmed like locusts to get a "piece of the action" and the potentially huge contingent fees that were likely to arise from the case. Interested in fees himself, but also rather disgusted at the performance of some of his colleagues of the bar, one attorney placed an ad in the town's weekly legal newspaper, whose readership was almost entirely lawyers. The ad suggested that any attorneys representing plaintiffs in the airline crash matter contact him in order to consolidate lawsuits against the airline, and that legal fees would be divided in proportion to the work performed. The ad was signed by the attorney and indicated his office address and telephone number. Was it proper for the attorney to place such an advertisement? response - incorrect A Yes, because the ad was not misleading. B Yes, because the lawyers will split the fees in proportion to work done. C No, because the attorney is soliciting business. D No, because the ad is in bad taste.

A. This is the best answer here because nothing in this advertisement violates the ABA Model Rules. The fact that the ad is not misleading is important because neither the ABA Model Rules nor the First Amendment protects misleading or deceptive advertising. (B) is incorrect because fee splitting is a separate issue; it does not affect the propriety of the ad. (C) is incorrect because the attorney here is not soliciting business, and also because the traditional ban on all solicitation is no longer constitutional. (D) is incorrect in that the ad is not patently in bad taste, and even if it were, it would probably be protected by the First Amendment unless it was misleading or overreaching.

An attorney and a real estate broker are friends. The real estate broker is of the opinion that the attorney is one of the best real estate lawyers in the community, and she recommends him to those persons seeking an attorney to close real estate transactions. The attorney needs the business and appreciates the real estate broker's recommendations. The attorney has given the real estate broker some of his business cards to give to individuals seeking a real estate lawyer. Currently, the attorney does not pay the real estate broker a referral fee. However, the attorney feels guilty for accepting the much-needed business and giving nothing in return. He has been considering a more formal arrangement whereby he would pay the real estate broker a modest $100 referral fee. In addition, he has been considering taking the real estate broker and her husband out to dinner as a way of expressing his appreciation for her past referrals. Which of the following is correct? response - incorrect A The attorney may represent clients referred by the real estate broker only if he continues his current practice of not paying the real estate broker a referral fee. B The attorney properly gave the real estate broker some of his professional business cards to give to individuals seeking a real estate lawyer. C The attorney may take the real estate broker and her husband out to dinner as a way of expressing his appreciation for her referrals. D The attorney may pay the real estate broker the modest $100 referral fee.

ABA Model Rule 7.2(b) provides that a lawyer must not give anything of value to a person for recommending the lawyer's services. Thus, (A) is correct, and (C) and (D) are incorrect. (B) is incorrect because a lawyer must not initiate an in-person contact with a nonclient, personally or through a representative, for the purpose of being retained to represent him for compensation. [ABA Model Rule 7.3]

For many years a lawyer has done business transactions work for a wealthy client. The client was recently injured in an automobile crash, and she has asked the lawyer to represent her as plaintiff in an action against the driver who injured her. The lawyer has taken some business cases to trial, but he has never handled a personal injury case. The lawyer would like to help his client and also generate some income. Which of the following would be an improper way for him to do so? A Take the case and, with the client's consent, associate a co-counsel who is competent in the field of personal injury law. B Refer the client to a competent personal injury lawyer and charge that lawyer a $1,000 forwarding fee. C Refer the client to a competent personal injury lawyer and charge the client a reasonable sum for the time spent in making the referral. D Take the case and, with the client's consent, undertake additional research to bring himself up to speed in the field of personal injury law.

B. A "forwarding fee" is another term for a "referral fee," and payments for referrals are prohibited. [ABA Model Rule 7.2(b)] (A) is proper because a lawyer may take on a case that he is not competent to handle if he obtains his client's consent to associate a lawyer who is competent to handle it. [Comment 2 to ABA Model Rule 1.1] (C) is proper because a lawyer may refer her client to another lawyer who is competent to handle the case. Making a sound referral can take a significant amount of time, especially if the referring lawyer needs to research the backgrounds of several lawyers with whom she is not personally familiar. It is appropriate for the referring lawyer to charge her client for the time spent making the referral, subject of course to the general rule on reasonableness. [ABA Model Rule 1.5] (As a practical matter, however, many lawyers would not charge a regular client for making such a referral.) (D) is proper because a lawyer may take on a case that he is not competent to handle if he undertakes the study necessary to provide competent representation. [Comment 2 to ABA Model Rule 1.1]

An attorney had an office in a small town located on the extreme western border of the state in which he was licensed to practice. The attorney received a retainer from a client, with the agreement that the attorney would use funds from the retainer for such things as filing fees when they came due. There was no federally insured bank or savings institution in the small town in which the attorney's office was located. The nearest such institution in the state was located in the county seat, which was over 60 miles away from the attorney's office and where the attorney and the client resided. Therefore, the attorney decided, with the client's consent, to put the client's money in his client trust account in a bank in a medium-sized city located just across the state line in a neighboring state. The account was fully insured by the federal government, but was not an interest-bearing account. The attorney is not licensed to practice law in that state. Was it proper for the attorney to place the client's money in an account in the neighboring state? response - incorrect A Yes, because retainer fees belong to the attorney and not to the client. B Yes, because the client consented to the deposit in the neighboring state. C No, because the attorney is not licensed to practice in the neighboring state. D No, because the funds were not placed in an interest-bearing account.

B. A client's funds are ordinarily deposited in an account where the lawyer's office is located, but they can be deposited elsewhere with the client's consent. [ABA Model Rule 1.15(a)] (A) is wrong because the client's funds appear to be an expense advance, not a "true retainer fee." (C) and (D) are wrong because there are no such requirements.

A judge is a loyal member of the alumni association of the women's college from which she was graduated. The 25th reunion of her graduating class is coming up, and she has been asked to participate in some activities designed to raise money for a gift from the class to the college scholarship fund. Which of the following activities would be improper for the judge to do? A Make a substantial personal donation to the class gift fund. B Telephone other members of her graduating class and urge them to make a donation to the class gift fund. C Serve on the scholarship fund committee, which devises the various fund-raising strategies. D Attend a fund-raising dinner for the class gift.

B. A judge may not personally solicit contributions for an organization other than from her family or certain other judges. [CJC Rule 3.7(A)(2)] Thus, (B) is improper. (A) is proper because Judge Jones, like anyone else, may contribute to any cause she likes. (C) is proper because a judge may assist an organization in planning fund-raising, although the judge may not actually participate in the fund-raising activity. [CJC Rule 3.7(A)(1)] A judge must not be a speaker or guest of honor at an organization's fund-raising event, but mere attendance at such an event is permissible. [CJC Rule 3.7(A)(4), comment 3] Thus, (D) is proper.

An attorney was formerly employed by a branch office of the Environmental Protection Agency ("EPA") as government counsel. In this capacity, he acted as chief counsel in several suits brought by the EPA involving chemical dumping into public waterways. Two years after leaving the employ of the EPA, the attorney was retained to represent a large corporation in a suit brought by the EPA alleging violations of certain EPA regulations regarding the dumping of chemical wastes. While with the EPA, the attorney was never directly involved in a case concerning this particular corporation. A different EPA branch office had exclusive responsibility for the drafting, promulgation, and enforcement of the regulations in question. Which of the following statements is correct? response - incorrect A The attorney may represent the corporation, but only with the consent of the EPA. B The attorney may represent the corporation, whether or not the EPA consents. C If the attorney represents the corporation without the consent of the EPA, he will be subject to discipline. D If the attorney represents the corporation, he will be subject to discipline, even if the EPA consents to the representation.

B. ABA Model Rule 1.11(a) prohibits a lawyer from representing a private client in a matter in which the lawyer participated personally and substantially as a public employee, unless the government agency in question gives its informed consent, confirmed in writing. The attorney here was neither personally nor substantially involved with any matter concerning the corporation, so he may now represent the corporation without obtaining the EPA's consent. Thus, (B) is correct, and (A), (C), and (D) are incorrect.

A district court judge heard through the "courthouse grapevine" that the district attorney was investigating corrupt practices in the courts and that the investigation focused on some as yet unascertained time in the past. The "rumor mill" also indicated that several judges and former judges were likely to be indicted for taking bribes to "fix" cases and to generate business for certain attorneys. The judge was alarmed at this news, and he telephoned a retired attorney, arranging to meet him for cocktails and dinner. At the restaurant, the judge slipped the maitre d' a 10-dollar bill to secure seating at a secluded corner booth. Over dinner, the judge told the retired attorney that he had accepted bribes in the past and that he did not know what he should do in light of the district attorney's investigation. The retired attorney advised the judge to do nothing. The judge picked up the $120 dinner tab, and the retired attorney thanked him for the fine meal. A month later, indictments were handed down against two sitting judges and three former judges. The judge was not among them, and it turned out that the period covered by the district attorney's investigation was prior to the judge's election to the bench. Six months after the indictments were announced, a member of the state appellate court died, and the governor announced that he was appointing the judge to serve the remaining three years of the justice's unexpired term. Must the retired attorney report his knowledge of corruption in office by the judge? response - incorrect A Yes, because the retired attorney never entered into an attorney-client relationship with the judge. B Yes, because the retired attorney's knowledge bears upon maintaining the integrity of the courts, which takes precedence over other ethical considerations. C No, because the retired attorney is retired from practice. D No, because the judge's disclosures to the retired attorney dealt with past crimes.

B. Although the retired attorney has retired from active practice, it is clear from the facts that the judge consulted him as an attorney, rather than as a friend; thus, the retired attorney may not reveal the judge's disclosures of past crimes. [See ABA Model Rule 1.6] Therefore, (D) is correct and (C) is incorrect. (A) is incorrect because an ongoing lawyer-client relationship need not be established for the rules of confidentiality to apply. (B) is an incorrect statement of a lawyer's ethical obligations, as no present fraud upon the tribunal or system of justice is contemplated, and ABA Model Rule 8.3 specifically states that matters considered confidential under Rule 1.6 are not included in the obligation to reveal judicial misconduct.

For three years, an attorney was a partner in a law firm. During that period, the attorney represented a client, Alpha, in obtaining a business loan from a bank. Alpha disclosed to the attorney a great deal of confidential information about his business and his personal assets. No other attorney in the law firm gained access to that confidential information. Recently, the attorney died. Shortly thereafter, Beta asked the senior partner of the law firm to represent him in a civil suit for serious personal injuries Beta suffered when he was run over by a delivery truck driven by one of Alpha's employees. Would it be proper for the senior partner to represent Beta? response - incorrect A Yes, because the information obtained by the deceased attorney about Alpha's assets has no effect on liability in a personal injury suit. B Yes, because neither the senior partner nor any other attorney in the law firm gained access to Alpha's confidential information. C No, because the conflict created by the deceased attorney's work for Alpha is imputed to the senior partner. D No, because the senior partner did not obtain Alpha's informed consent, confirmed in writing.

B. It would be proper for the senior partner to represent Beta because neither the partner nor any other lawyer in the firm gained access to Alpha's confidential information. Even though Alpha's bank loan is not substantially related to Beta's personal injury suit, the confidential information that the deceased attorney got from Alpha may well become important in Beta's suit. Knowing the extent and nature of Alpha's assets could be of great value to counsel for Beta in advising Beta whether to settle or in collecting on a judgment against Alpha. Here, however, the deceased attorney was the only attorney who gained access to Alpha's financial information, and the attorney is now dead. Under ABA Model Rule 1.10(b), the senior partner may represent Beta, because neither he nor any other attorney remaining in the firm had access to Alpha's confidential information. (A) is wrong because it ignores the issue presented by the confidential information. (C) is wrong because ABA Model Rule 1.10(b) creates an exception to the ordinary rule that confidential information gained by one lawyer in a firm is deemed to be known by all lawyers in the firm. (D) is wrong because ABA Model Rule 1.10(b) allows the senior partner to serve even without the consent of Alpha.

jurisdiction and venue, the case had to be filed and tried in State B. The written fee agreement between the client and the attorney provided that: (1) The attorney would assume full responsibility for the case as lead lawyer; (2) The client would pay the attorney 40% of the net recovery after deduction of litigation expenses; (3) The attorney would associate a State B lawyer to serve as trial counsel in State B; (4) The State B attorney would assume responsibility only for his work as trial counsel; and (5) The attorney would pay the State B attorney an appropriate portion of the 40% contingent fee. Would it be proper for the attorney to split his fee with the State B attorney under the circumstances described above? t A No, because the attorney is not admitted in State B. B No, because the share that each lawyer will receive was not disclosed in the written fee agreement. C Yes, because the State B attorney was assuming responsibility for his work as trial counsel. D Yes, because there was a written fee agreement.

B. It would not be proper for the attorney to split his fee with the State B attorney because the written fee agreement with the client does not comply with the ABA Model Rules. ABA Model Rule 1.5(e) allows a lawyer to split a fee with a lawyer who is not in his firm if: (i) the total fee is reasonable; (ii) the split is in proportion to the services rendered by each lawyer, or in some other proportion if each lawyer assumes joint responsibility for the matter; and (iii) the client agrees to the split in a writing that discloses the share that each lawyer will receive. Here, the written fee agreement did not specify the share that each lawyer will receive; thus, a fee split between the attorney and the State B attorney would be improper. (A) is wrong because there is no requirement that a lawyer be licensed in the same state as the attorney with whom he is splitting a fee. (C) is wrong because the written fee agreement with the client did not indicate the share that each lawyer will receive, and thus the agreement was improper regardless of whether the State B attorney was assuming responsibility for his work. (D) is wrong because even though there was a fee agreement, it did not comply with the ABA Model Rules.

A lawyer practices in State A. State A's rules of legal ethics depart from the ABA Model Rules in one significant respect: State A has no "financial injury" exception to the lawyer's duty of confidentiality. Thus, when a State A lawyer learns in confidence that her client is about to use her legal services to inflict serious financial injury on someone, the lawyer may withdraw, but she must not reveal what she learned in confidence. The lawyer limits her practice to federal securities law, and she regularly appears before the Securities and Exchange Commission ("SEC"). One of her major clients is a company that makes and sells cotton textiles. The company's shares are traded on the New York Stock Exchange and in securities matters the company is regulated by the SEC. While working on an SEC registration statement for the company, the lawyer learned in confidence that three of the company's top executives were cooperating in a scheme to loot the company of millions of dollars. If their scheme continues, it could drive the company into insolvency. The lawyer alerted the chief legal officer of the company to the situation, but he did nothing. She then alerted the chief executive officer, who also did nothing. Finally, she alerted the six outside members of the board of directors, but they too failed to act. In disgust, the lawyer withdrew from the matter and vowed never again to represent the company. Must the lawyer now tell the SEC about the scheme? A Yes, the SEC's regulations under the Sarbanes-Oxley Act require her to alert the SEC if her other efforts have proven fruitless. B No, the SEC's regulations give her discretion to either reveal or not reveal the matter to the SEC. C Yes, because the shareholders could be seriously financially injured if the scheme continues. D No, because State A's legal ethics rules do not allow her to reveal confidential information in this situation.

B. The SEC's regulations under the Sarbanes-Oxley Act permit, but do not require, a securities lawyer to reveal a client's confidential information to the SEC when the lawyer reasonably thinks that doing so is necessary to prevent or rectify a securities act violation (or similar law violation) that is likely to cause substantial financial injury to the client or its shareholders. (A) and (C) are wrong because the regulation permits, but does not require, the lawyer to alert the SEC. (D) is wrong because the SEC regulations purport to preempt any inconsistent state ethics rules. [See 17 C.F.R. §§205.1, 205.6(c)] (It remains to be seen whether courts will uphold the SEC's effort to preempt the field and override inconsistent state ethics rules.)

An attorney was representing the plaintiff at a bench trial of a civil action pending before a judge. Midway through the plaintiff's case-in-chief, the judge called the attorney into his chambers. The judge told the attorney that he thought the attorney's case was very weak, but that he could be mistaken because he was distracted by money troubles. The judge went on to say that if he could get a $50,000 loan, he would feel much better. The attorney responded that he would be happy to loan the judge $50,000 to help him out as a friend. Later that afternoon, a messenger delivered an envelope containing $50,000 in cash to the judge's chambers. No mention was made of a promissory note, a repayment date, or an interest rate. Two days later, the plaintiff settled his lawsuit so the judge never had to decide the case. Three months later, the judge repaid the $50,000 to the attorney, together with interest at the market rate. Is the attorney subject to criminal liability for lending the money to the judge? A Yes, if it is proven that the judge intended to induce the attorney to make the loan in return for a decision in favor of the plaintiff. B Yes, if it is proven that, in making the loan, the attorney intended to induce the judge to decide the case in favor of the plaintiff. C No, because as the matter turned out, the judge never had to decide the plaintiff's case. D No, because the judge repaid the loan with interest.

B. The attorney is subject to criminal liability if he intended to induce the judge to decide the case in the plaintiff's favor. The common law crime of bribery consists of the corrupt payment or receipt of anything of value in return for official action. The $50,000 loan was obviously a thing of value. Thus, if the attorney intended the loan as an inducement to the judge to decide the case in favor of the plaintiff, then the attorney is guilty of bribery. (A) is wrong because in deciding whether the attorney is subject to criminal liability, it is the attorney's intent that counts, not the judge's intent. (C) is wrong because the attorney's crime was complete when he gave the loan, even though the judge never had to decide the case. (D) is wrong because bribery does not require an outright gift; a $50,000 loan is a thing of value, especially a loan with no repayment date, no promissory note, and no interest specified.

A client hired a lawyer to draft a will for him. The client willed his entire estate to a 43-year-old widow. The client told the lawyer in confidence that he was neither a relative nor a friend of the widow. The client explained that he felt a moral obligation to the widow because he had killed her husband, and he had never become a suspect or confessed his sin to anyone. One day after signing the will, the client committed suicide. In due course, all of the client's assets were distributed to the widow, and the probate court closed his estate and discharged his executor. The lawyer never told the widow or anyone else that the client had confessed to killing the widow's husband. Now, a few years later, an enthusiastic young prosecutor is charging an innocent man with murdering the widow's husband in the first degree with aggravating circumstances, and the prosecutor is seeking the death penalty. May the lawyer voluntarily tell the innocent man's defense counsel what his client told him in confidence about killing the widow's husband? A Yes, the lawyer not only may, but he must, tell the defense counsel what the client told him. B Yes, the lawyer may tell, but he would not be subject to discipline if he decides not to do so. C No, the lawyer would be subject to discipline if he told defense counsel because the attorney-client privilege survives the death of the client. D No, because the client's confidential confession to the lawyer would be inadmissible hearsay if offered against the prosecution in the murder trial.

B. The controlling doctrine in this case is the lawyer's ethical duty of confidentiality, not the attorney-client privilege. The lawyer needs to know whether he can voluntarily reveal the client's confession, not whether he would be forced to do so if he were put on the witness stand in a court. ABA Model Rule 1.6(b)(1) states the applicable exception to the ethical duty of confidentiality: A lawyer may reveal confidential information if the lawyer reasonably believes that doing so is necessary to prevent reasonably certain death or substantial bodily harm. One might quibble whether the innocent man's death is "reasonably certain" when his trial has not even started, but surely the ethics rule should not be read to require the innocent man to order his last meal before being loosed from the executioner's grip. (A) is wrong because ABA Model Rule 1.6(b)(1) gives the lawyer discretion to reveal the client's confession; the Rule does not force him to do so. [See comment 15 to ABA Model Rule 1.6] (A few states go farther and require disclosure to prevent death or substantial bodily harm, but they are a small minority.) (C) is wrong for two reasons. First, the applicable doctrine is the ethical duty of confidentiality, not the attorney-client privilege. Second, even if the privilege were the applicable doctrine, who could claim it in this situation? The client cannot because he is dead. The client's executor cannot because the client's estate was closed and the executor was discharged. The lawyer cannot claim it because a lawyer's right to claim the privilege is only derivative from the client. (D) is wrong for two reasons. First, the admissibility of this hearsay is irrelevant to the ethics issue. Second, the client's confession would likely be admissible if offered by the innocent man against the prosecution because it is a declaration against penal interest by an unavailable declarant, and the client's will and suicide are independent evidence of the confession's trustworthiness. [See Fed. R. Evid. 804(b)(3); see also Chambers v. Mississippi, 410 U.S. 284 (1973)—due process violation where another man's confession was excluded in a murder trial]

A judge sits on a federal appellate court. He and two other federal judges heard a diversity of citizenship case in which they were required to interpret a state statute concerning the marital communications privilege. The judge's two colleagues wrote the majority opinion, in which they concluded that the statute gives only the witness-spouse the right to claim the privilege. The judge wrote a vigorous and scholarly dissent, arguing that the statute gives both spouses the right to claim the privilege. Later, a state senator introduced a bill to amend the statute to reflect the judge's position. The state senate invited the judge to testify about the public policy reasons for giving both spouses the right to claim the privilege. May the judge testify? A Yes, but only if the two judges who wrote the majority opinion are also allowed to testify. B Yes, because a judge may engage in activities designed to improve the law. C No, because a judge must not become involved in politics, subject to certain exceptions that do not apply here. D No, because a judge is not allowed to make public statements about disputed propositions of law, except when acting in his judicial capacity.

B. The judge may testify at a public hearing in connection with matters concerning the law. [CJC Rule 3.2(A)] (A) is wrong because there is no rule requiring "equal time." (C) is wrong because it is overbroad. The general rule against judicial involvement in politics limits only some types of political activities, not including legislative testimony. [CJC Canon 4] (D) is wrong because, with respect to issues that are likely to come before the court, a judge is prohibited from making pledges, promises, or commitments that are inconsistent with the impartial performance of his duties. [CJC Rule 4.1(A)(13)] That Rule does not apply here because the judge's testimony, which would be designed to improve the law, would not constitute a promise that is inconsistent with the performance of his adjudicative duties.

A lawyer is defending a marine supply company in a civil action brought by the state attorney general under a statute that makes it a civil offense for any person or business entity to bribe or give a kickback to a state official. The statute authorizes fines of up to $100,000 per transaction for any violation. The marine supply company has a strict corporate policy that prohibits its employees from bribing or giving kickbacks to anyone. Employees who violate the policy are subject to immediate discharge and are required to indemnify the marine supply company for any loss it suffers as a consequence of the violation. The attorney general has noticed the depositions of dozens of the marine supply company's employees. One of these employees, prior to his recent retirement, was the sales manager of the marine supply company. The lawyer met with this employee to prepare him for his deposition. At the outset of the interview, the lawyer agreed to represent the employee without charge, and the lawyer told the employee that anything said between them would be confidential. During the interview, the lawyer asked the employee whether he had ever bribed any state officials. The employee confessed that he had, but said it had been necessary because all of the company's competitors were doing it, too. What course of action may the lawyer pursue at this point? A Withdraw from the case and inform the attorney general what the employee said. B Withdraw from the case and keep the employee's statement in confidence. C Withdraw from representing the employee and inform the marine supply company what the employee said. D Continue in the case, inform the marine supply company what the employee said, and advise the marine supply company to seek prompt settlement.

B. The lawyer may withdraw from the case and keep the employee's statement in confidence. When an organization is the lawyer's client, the lawyer owes a duty of loyalty to the organization. When the interests of the organization and its constituents conflict, the lawyer should remind the person that the lawyer represents the organization and not the person. It would be appropriate for the lawyer to remind the person that communications between them may not be protected by the attorney-client privilege, and that the person may want to obtain independent counsel. [ABA Model Rule 1.13] Here, the lawyer should not have asked the question unless he was prepared for an affirmative answer. He should have known before asking that if the answer was yes, the employee's interests and the marine supply company's interests would conflict; thus, the lawyer should not have offered to represent the employee, and certainly should not have promised to keep the employee's statements in confidence. (Note that the lawyer could be subject to discipline for this conduct.) The issue here, however, is what course of action the lawyer may now take. Now that the lawyer has agreed to represent the employee, and the employee has confessed in confidence, the only thing the lawyer can do is withdraw from the matter entirely and keep the employee's confession in confidence. [See ABA Model Rule 1.9] (A) is wrong because he would violate the employee's confidence by disclosing the confession to the Attorney General. (C) and (D) are wrong because revealing the employee's confession to the marine supply company would also violate the employee's confidence.

A lawyer regularly represents a manufacturer of electric kitchen appliances. One morning the president of the manufacturing company called the lawyer and asked if the lawyer had seen the newspaper story about a woman who was electrocuted when she opened the door of her dishwasher. The company president stated that he believed the dishwasher was one that his company had manufactured. The company president also stated that he found some quality control records from that period which reflected that some dishwashers left the plant without proper testing. He continued that the records should have been shredded, but somehow had been overlooked, and said that he intended to send the records to the shredder immediately unless the lawyer told him that he could not. Must the lawyer advise the president to keep the records? A Yes, unless the company has a clearly established policy of shredding quality control records after two years. B Yes, because the records have potential evidentiary value if the company gets sued. C No, because at this point there is no litigation pending against the company respecting this matter. D No, unless it was certain that the company was the manufacturer of the dishwasher in question.

B. The lawyer must advise the president to keep the records because the records have potential evidentiary value if the company is sued. A lawyer must not counsel or assist a person to destroy material that has "potential evidentiary value." [ABA Model Rule 3.4(a)] Although it is not certain that the company manufactured the dishwasher in question, the president said he was "pretty sure that it was one of ours." Furthermore, it is not certain that the company will be sued if it was one of their washers, but the chances are good that it will be. If commencement of proceedings can be foreseen, the documents have potential evidentiary value and cannot be destroyed. [Comment 2 to ABA Model Rule 3.4] (A) is wrong because even if the company's records retention program called for the routine shredding of these records long ago, they were not shredded then, and they have potential evidentiary value now. (C) is wrong because commencement of proceedings is foreseeable. (D) is wrong because the standard is backward. The records should be preserved until the company is certain that the dishwasher in question was not manufactured by the company.

A lawyer practices real estate law in an old-fashioned jurisdiction in which almost every real estate transaction requires the services of one or more lawyers. The lawyer is also licensed by the state as a real estate broker. The lawyer conducts her law practice and her real estate brokerage business in a single office, using one secretary and one paralegal as her support staff. The lawyer specializes in small, relatively old apartment buildings (from 4-12 units) that are not in peak condition. They make good investments because they can be bought cheap, fixed up, and leased at favorable rates. The lawyer generates most of her business by discovering who owns a likely looking building and then making a face-to-face pitch to the owners, trying to interest them in selling and using her to find a buyer. After the owners sign her up as their real estate broker, the lawyer lets them know that she can also do the necessary legal work—the title search, the financing documents, the land transfer documents, and the like. Is the lawyer subject to discipline? A Yes, because a person who is engaged in full-time law practice must not conduct a related business from a single office. B Yes, because a person who offers legal services along with real estate brokerage services must not initiate face-to-face contact with persons to drum up business. C No, because the lawyer's real estate brokerage services are ancillary to her law practice, and the two operations are conducted from a single office. D No, so long as her face-to-face pitch to the owners of apartment buildings is truthful and not misleading.

B. The lawyer's real estate brokerage business is a "law-related service" within the meaning of ABA Model Rule 5.7, and the lawyer offers her real estate brokerage services "in circumstances that are not distinct from" her provision of legal services. [See ABA Model Rule 5.7(a)(1)] That means that she must follow the rules of legal ethics in her real estate brokerage work as well as her law work. [Id.] One of the legal ethics rules forbids a lawyer from initiating in-person contact with a target of solicitation when a significant motive for doing so is the lawyer's pecuniary gain. [See ABA Model Rule 7.3(a)] Therefore, the lawyer must not initiate face-to-face contact with potential real estate clients to interest them in using her brokerage services. [See comment 10 to ABA Model Rule 7.3] (A) is wrong because it overstates the rule expressed in ABA Model Rule 5.7. (C) is wrong because it turns ABA Model Rule 5.7 on its head—because the lawyer is offering her ancillary service in circumstances that are not distinct from her legal service, she must follow the legal ethics rule for both kinds of service. (D) is wrong because the lawyer's face-to-face pitches violate the no-solicitation rule even if her statements are truthful and not misleading. [Compare ABA Model Rule 7.1 with ABA Model Rule 7.3(a)]

A Hollywood movie producer was charged under a criminal statute for unfair trade practices, and now faces a civil claim under the same statute. The producer retains an attorney to represent him in both suits. The attorney is a nationally known defense attorney who has represented many famous people. Most recently, he defended a celebrity in a notorious murder case that held the country rapt for several weeks. The attorney explains to the producer that the representation is very complex and would take a majority of his time for several months. Given the attorney's steep hourly rate, the producer's legal fees would likely be around $1 million. The producer is short on cash and makes the following proposal: If the attorney will represent him in both the civil and criminal suits, the producer will produce a movie based on the attorney's most famous past cases, told from the attorney's viewpoint. The attorney would have complete creative control and would be entitled to all of the movie's profits, which could be anything from $0 to $100 million. The producer had his personal attorney draw up a proposal to this effect and submitted it to the attorney. Assuming that the attorney receives any consent necessary from his former clients who might be portrayed in the movie, is this proposed arrangement proper? A Yes, but only if the payment from the movie profits is for the civil suit only. B Yes, but only if the ultimate amount paid to the attorney is not excessive in light of the work done. C No, because any amount over $1 million is clearly excessive, and this arrangement could be worth $100 million. D No, because a lawyer must not acquire media rights to a story concerning the lawyer's representation of a client.

B. This arrangement is proper if the fee is reasonable under the circumstances. As long as the fee paid does not turn out to be excessive, taking into account the attorney's risk of not being paid, the delay in payment, etc., this arrangement is acceptable. An attorney may enter into a business relationship with a client, provided certain safeguards, such as an opportunity to consult with independent counsel, are used. [ABA Model Rule 1.8(a)] (A) is wrong because although it is improper to use a contingent fee in a criminal case, this fee is not a contingent fee. Whether the attorney gets paid does not depend on the outcome of the case; it depends on how well the movie does. (C) is wrong because it is not clear that any amount over $1 million is excessive, given that the attorney is risking that he will be paid nothing and is delaying payment by a substantial period of time. (D) is wrong because it misstates or incompletely states the Rule. The Rule is that prior to the conclusion of the representation of the client, an attorney cannot acquire media rights to a story based substantially on information relating to the representation. In this case, the movie does not relate to the current representation, rather it involves past representations in the attorney's career. While the attorney could not disclose any information related to those representations without the clients' consents, the acquisition of such media rights is not improper.

An attorney who is a member of the state bar and a judge who sits on the district court set up a probate workshop. The announced purpose of the workshop was to educate laypersons as to probate procedures, and thereby enable them to file their own papers and avoid the expense of obtaining counsel for the probate process. The tuition was a modest $50 per student, which barely covered the expenses of books and forms provided to the students. The attorney and the judge donated their time and received no remuneration. Each student was required to sign a paper stating that the $50 tuition fee established an attorney-client relationship between the attorney conducting the workshop and the students. The paper further stated that all attorneys connected in any way with the workshop were not liable for any damages that might be incurred by students as a result of pro se filings made in accordance with, or at variance from, the instructions provided during the workshop. The paper was signed by the attorney and the judge. Is the attorney subject to discipline? A Yes, because she has participated in the forcing of an attorney-client relationship upon the students. B Yes, because an attorney may not limit her malpractice liability by contract. C No, because the attorney is helping the public to avoid excessive legal fees through her participation in the workshop. D No, because the clients agreed to the contract as a precondition for obtaining the benefits offered by the workshop.

B. ABA Model Rule 1.8(h) makes it clear that an attorney is subject to discipline for attempting to prospectively limit malpractice liability by contract with a client. The ABA Model Rules provide an exception if the client secures independent counsel in making the agreement. That exception does not apply here. (A) is incorrect because the "clients" are free to go elsewhere for representation and no relationship is "forced" on them. (C) is incorrect because high-sounding rhetoric and purpose do not change the impropriety of attempts to limit malpractice liability by contract. (D) is incorrect because the clients were not independently represented in making agreements to the contract.

A corporation is named as the defendant in an employment discrimination suit. The corporation's attorney has scheduled the taking of a deposition of one of the corporation's employees who is in poor health, as a precaution in case the employee should die before trial. The employee is not an officer or shareholder in the corporation, and he is not a party to the lawsuit. He is in charge of the corporation's personnel department, and he is responsible for ensuring that the corporation's hiring practices comply with the laws against employment discrimination. Without seeking the consent of the corporation's attorney, or even telling him, the plaintiffs' attorney had lunch with this employee several days before the deposition, and on that occasion the plaintiffs' attorney pumped the employee for information relevant to the lawsuit. When the corporation's attorney learned what had happened, he telephoned the plaintiffs' attorney and called him a slimy, mud-sucking shyster. Which of the following is correct? response - incorrect A The corporation's attorney's conduct was proper. B The plaintiffs' attorney's conduct was proper. C The plaintiffs' attorney is subject to discipline because he should not have talked with the employee about the case without the consent of the corporation's attorney. D The plaintiffs' attorney is subject to discipline because he talked with a deposition witness about the subject of the litigation before the deposition was taken.

C. (A) is incorrect because lawyers should treat all participants in a proceeding with courtesy, respect, and cooperation. [See Restatement §106] (D) is incorrect because there is nothing that prohibits a lawyer from talking with a deposition witness about the case before a deposition. However, a lawyer must not communicate about a case with a person he knows to be represented by a lawyer, without first getting the lawyer's consent. [ABA Model Rule 4.2] Thus, (C) is correct and (D) is incorrect. Also, Comment 7 to ABA Model Rule 4.2 states that, in the case of a represented organization, a lawyer for one party may not communicate concerning the matter with persons whose acts or omissions in connection with the matter may be imputed to the organization for purposes of civil or criminal liability. Here, the corporation's employee fits the description of such a person. Thus, (B) is incorrect.

An attorney recently opened his solo law practice in a small town. His practice is fairly evenly divided between civil litigation and criminal defense. The Superior Court has just appointed the attorney to represent two defendants, who will be tried jointly for their alleged kidnapping and brutal murder of nine local school children. Which of the following is not a valid reason for the attorney to decline the appointment? A He believes that to represent the two defendants will take so much time away from his newly opened practice as to impose an unreasonable financial burden on him. B He believes that one defendant coerced the other defendant into helping kidnap and kill the children. C He believes many of his potential clients will be outraged if he represents the two defendants. D He believes that confidential information he received when representing one of the prosecution's key witnesses will be useful in impeaching that witness's credibility.

C. A lawyer can be disciplined for trying to avoid a court appointment without good cause. [ABA Model Rule 6.2] The reason stated in (C) is not an acceptable reason for declining the appointment; a lawyer has a duty to represent his fair share of indigent or unpopular clients. [Comment 1 to ABA Model Rule 6.2] (A) is a legitimate reason for declining an appointment. A lawyer is permitted to turn down a court appointment if it "is likely to result in an unreasonable financial burden." [ABA Model Rule 6.2(b)] (B) is also a legitimate basis for declining appointment because a lawyer may turn down a court appointment if it is likely to cause the lawyer to violate a rule of professional conduct. [ABA Model Rule 6.2(a)] If the two defendants are to be tried jointly, and if one defendant did coerce the other defendant into helping with the kidnapping and killing, there is a sharp conflict of interest between the two defendants. [See ABA Model Rule 1.7(a)] It would be an ethical violation to represent co-defendants with conflicting interests (consent will not solve the conflict); thus, the attorney can decline the appointment on this ground. (D) raises another conflict of interest that would justify the attorney in declining the appointment. Had he not gained confidential information from the prosecution's witness, he might have discovered that information independently and been able to use it to impeach the witness. As it stands, however, his ability to impeach is constrained by his duty not to use the confidential information to the disadvantage of the witness, his former client. [ABA Model Rule 1.9(c)

A juvenile charged with auto theft, which is a felony, hires an attorney to represent him. Before the case comes to trial, his attorney confers with the prosecutor who has been assigned to prosecute the case. The prosecutor suggests to the juvenile's attorney that, because her client is a juvenile, the charge might be reduced to "joyriding," a misdemeanor. The juvenile's attorney flatly refuses without explanation, unreasonably, in the prosecutor's opinion. It would be improper conduct for the prosecutor to: A Ask the court to dismiss the auto theft charge and prosecute the juvenile for joyriding. B Prosecute the auto theft charge. C Send an investigator to talk to the juvenile about the offer to lower charges. D Remind the juvenile's attorney of the duty to communicate settlement offers to the client.

C. A lawyer must not communicate about a matter with a person the lawyer knows is represented by counsel, unless that person's counsel consents or the law authorizes the communication. [ABA Model Rule 4.2] Thus, (C) is clearly improper. Because it is within a prosecutor's discretion to request that the court prosecute on a lesser offense, (A) is proper. (B) is proper because there is no indication that the juvenile could not legitimately have been prosecuted for auto theft. (D) is proper because the communication is between counsel, and defense counsel must promptly inform her client of a proffered plea bargain "unless the client has previously indicated that the proposal will be acceptable or unacceptable or has authorized the lawyer to accept or to reject the offer." [See Comment 2 to ABA Model Rule 1.4]

An attorney in solo practice published a brochure regarding what one should do when injured. The brochure contains accurate, helpful information about obtaining proper medical treatment, recording details of the accident, notifying insurance companies, not making harmful statements, and the like. The attorney's name, address, and telephone number are printed on the brochure's cover. One afternoon, the attorney saw a pedestrian knocked down in a crosswalk by a hit-and-run driver. He and another bystander called 911 and gave the pedestrian emergency first aid until an ambulance arrived. The next day, the attorney visited the pedestrian in the hospital and gave the pedestrian a copy of his brochure. Which of the following is correct? response - incorrect A The attorney is subject to discipline, both for publishing the brochure and for giving the brochure to the pedestrian in the hospital. B The attorney is subject to discipline for publishing the brochure. C The attorney is subject to discipline for giving the pedestrian a copy of the brochure at the hospital. D The attorney's conduct was proper because the brochure's contents are neither false nor misleading.

C. A potential problem arises when the content of a brochure is improper or when the printed material is used as part of an act of solicitation. Here, the content was proper, but when the attorney gave the brochure to the hospitalized pedestrian, he crossed the line into impermissible in-person solicitation. [See ABA Model Rule 7.3; see also Ohralik v. Ohio State Bar Association, 436 U.S. 447 (1978)—noting the particular potential for undue influence and overreaching in the hospital context, where individuals are often especially vulnerable] (A) and (B) are incorrect because publishing the brochure, in and of itself, is not problematic, just as the act of printing business cards is fully permissible. (D) is incorrect because the truthfulness of the brochure's contents does not counteract the attorney's impropriety in providing his contact information during a hospital visit.

The attorney general's office does not include any lawyers who are skilled in the field of condemnation law (the law of eminent domain). Consequently, whenever the state wants to use its power of eminent domain to condemn some private property for a public use, the attorney general must hire a private law firm to represent the state in the condemnation proceedings. In contrast to the paltry fees that the state pays to appointed defense counsel in criminal cases, the attorney general pays quite handsomely for condemnation work. The attorney general is a partisan political position that is filled by a contested election every four years. A large state law firm limits its practice to condemnation law. The founding partner is an 87-year-old multimillionaire who remains active on the firm's management committee. When it is time to elect a new attorney general, the partner makes large donations from his personal wealth to each candidate who has any reasonable chance of becoming the next attorney general. The other members of the firm's management committee know about the partner's contributions, and they have formally and informally expressed the firm's thanks for helping the firm obtain future appointments by the attorney general. May the firm accept an appointment from the new attorney general to represent the state in a condemnation case? response - incorrect A Yes, because the partner makes his contributions from his personal wealth, and he has a constitutional right to participate personally in the political process. B Yes, because the partner's personal political contributions cannot be imputed to the law firm. C No, because a lawyer or law firm must not accept appointed legal work from a governmental official after making a political contribution for the purpose of obtaining such work. D No, because to accept such an appointment would create an appearance of impropriety in light of the partner's political contributions.

C. ABA Model Rule 7.6 prohibits "a lawyer or law firm" from accepting an appointed legal engagement if "the lawyer or law firm" makes a political contribution "for the purpose of obtaining or being considered for" that kind of legal engagement. The tricky part of this question is whether a political contribution by one of the firm's lawyers ought to bar the entire firm from taking subsequent appointments. Neither the Rule nor its comments speak directly to that point, but the purpose of the Rule would be served by imputing one lawyer's contribution to the entire firm, just as a conflict of interest would be imputed under ABA Model Rule 1.10(a). To adopt the opposite position would make ABA Model Rule 7.6 too easy to evade—the firm could simply ask its lawyers to make "pay-to-play" contributions from their own pockets. Using a different theory, at least the founding partner's colleagues on the management committee should be barred from accepting appointments from the attorney general because they knew about the partner's political contributions and thanked him rather than stopped him. [See ABA Model Rule 5.1(c)—ratification or acquiescence by managing lawyers] (B) is wrong for the reasons stated above. (A) is wrong because the partner's practice of contributing generously to all candidates who have a reasonable chance to win the attorney generalship demonstrates that his purpose is to secure business for the firm, not to participate legitimately in the political process. (D) is not as good as (C) because (D) relies on the outdated "appearance of impropriety" rubric of the old ABA Model Code, Canon 9. That rubric was cast aside in the ABA Model Rules—seeking to avoid even the "appearance of impropriety" is useful in a person's own moral creed, but it is too amorphous to be useful in a professional code of conduct.

Attorneys Alpha and Beta have been law partners for six years. Beta was suspended from practice for one year based on an intentional tax law violation. Alpha took over Beta's clients when his suspension went into effect. Shortly before his suspension, Beta had negotiated a $30,000 personal injury settlement with an insurance company on behalf of his client. Two weeks after the settlement was reached, the insurance company sent a $30,000 check to the law offices. By this time Beta's suspension had gone into effect. Alpha placed the check in the proper firm account and confirmed the amount of the fee with Beta's client. Alpha then promptly forwarded a $20,000 check to the client and a $10,000 check to Beta, the latter check representing Beta's one-third contingent fee. Is Alpha subject to discipline? A Yes, because she should have held the $10,000 in the client trust account until Beta's suspension had ended. B Yes, because a lawyer is prohibited from sharing legal fees with a nonlawyer. C No, because Beta earned the fee prior to his suspension. D No, because the $10,000 belonged to Alpha's firm, and she could do anything she wished with it, including sending it to Beta as a gift.

C. Alpha is not subject to discipline because Beta earned the fee prior to his suspension. Despite his suspension, Beta is entitled to the fees he earned while he was still lawfully practicing law. It is true that a lawyer is prohibited from aiding a nonlawyer in the unauthorized practice of law, but here Beta is not practicing law, and Alpha is merely forwarding his previously earned fee. (A) is wrong because Beta need not wait until he is reinstated to collect a fee he earned prior to his suspension. (B) is wrong because Beta is not a nonlawyer, and Alpha is not splitting legal fees with him in any case. Alpha is merely transmitting Beta's own money to him. (D) is wrong because the money does not belong to Alpha, and even if it did, she cannot do anything she wishes with it. For example, there are rules prohibiting the sharing of legal fees with nonlawyers.

A car owner is insured under an auto liability policy issued by a nationally known insurance company. The policy requires the insurance company to provide a lawyer to defend the car owner, and it requires the car owner to cooperate in the defense. The car owner had an accident and was sued. In a sworn statement to the insurance company's investigator, the car owner told a story that showed he was clearly not at fault. Based on that story, the insurance company rejected plaintiff's offer to settle the case for a modest sum. The insurance company hired an attorney to represent the car owner at the trial of the case. Shortly before trial, the car owner told the attorney in confidence that he had lied to the investigator, and he recounted facts that showed he was clearly at fault in the accident. The attorney realized that under the applicable state law, the car owner's falsehood was a breach of the "cooperate in the defense" clause, and that it relieved the insurance company of any further duties to the car owner. At this juncture, what must the attorney do? A Promptly advise the insurance company of the situation and carry out its instructions as to how to dispose of the matter. B Promptly advise the car owner that his best interests will be served by reverting to the story he told originally to the insurance investigator. C Promptly seek the court's permission to withdraw from the matter, without revealing the car owner's confidential statement to anyone. D Promptly advise the car owner of the legal consequences of his false statement, and continue representing him and the insurance company in the matter as best he is able in the circumstances.

C. Ordinarily, an attorney can act for both the insured and the insurance company because their interests are only potentially in conflict. But here they have come into present, actual conflict. The insurance company's attorney cannot adequately represent the car owner without harming the insurance company, and he cannot protect the insurance company's interests without harming the car owner. Therefore, he must seek the court's permission to withdraw from the case entirely, and he must not reveal the car owner's confidential statement. [See ABA Model Rules 1.6, 1.7, 1.16(a)(1)] In a case very much like this one, the Seventh Circuit held that the attorney "should have refused to participate further in view of the conflict of interest" between the insured person and the insurance company. [State Farm Mutual Automobile Insurance Co. v. Walker, 382 F.2d 548 (7th Cir. 1967); and see ABA Informal Op. 1476 (1981)] Thus, (C) is correct, and (A) and (D) are incorrect. (B) is incorrect because of the conflict of interest and because the attorney may be advocating perjury.

An attorney represents a man who has pleaded guilty to a burglary charge. The accused told the attorney during one of their confidential conversations that this is the fourth time he has been busted for burglary—one other time in this state and two times in the neighboring state. The accused could be facing a mandatory 5-10 years in prison as a repeat offender, but he and the attorney are hoping that his pleading guilty will result in a shorter sentence. Unknown to the attorney or the accused, a glitch in the state computer files resulted in a pre-sentencing report that did not pick up either the accused's in-state or out-of-state prior convictions. At sentencing, the judge states, "Normally, I throw the book at young men like you who have no respect for the property of others. However, because I see you have no prior criminal record, I think you deserve another chance. I sentence you to two years' probation and 300 hours of community service." Both the attorney and the accused remain silent. Is the attorney subject to discipline? response - incorrect A Yes, because failure to speak out when one knows that the court is operating on false information is the equivalent of affirmative misrepresentation. B Yes, because this is a sentencing hearing rather than a trial. C No, because the mistake did not originate with the attorney or the accused. D No, because the attorney could not reveal the confidential information even if he had been asked directly.

C. The attorney is not subject to discipline for failure to correct the court's mistake because the mistake did not originate with the attorney or his client. In this setting, the attorney would have a duty to speak out if something he or his client had done had given the court a false impression, but otherwise, he cannot breach his duty of confidentiality to the disadvantage of his client. [See ABA Model Rules 1.6(a), 3.3] Thus, (A) is incorrect. The fact that this is a sentencing hearing does not affect the attorney's duties in any way. This is not an ex parte proceeding (which would be a much thornier issue). Thus, (B) is incorrect. (D) is incorrect because had the judge asked the attorney directly whether this was his client's first offense, the attorney would have had to respond truthfully or have asked to withdraw (which would also have given the judge his answer). To answer otherwise would be perpetrating a fraud on the court.

An attorney worked at the United States Department of Labor and was responsible for compiling certain corporate safety records into an annual report containing the accident statistics. The report is used internally and in discussions with companies, but it is not distributed to the general public. However, a person may obtain a copy of the report by filing a formal request under the Freedom of Information Act. During the last three years, Company A has had more accidents than any of the other reporting companies. Six months ago, the attorney left the Labor Department and took a job with a private law firm. Recently, a person came to the attorney seeking representation in a suit against Company A for injuries he sustained while working at Company A's factory. The attorney agreed to represent the client. Is the attorney subject to discipline? A Yes, because he obtained relevant information about Company A while working as a government attorney. B Yes, because the attorney did not obtain the consent of the Department of Labor. C No, because the information is available by formal request under the Freedom of Information Act. D No, if the attorney does not use the information obtained while employed as a government attorney to the material disadvantage of Company A.

C. The attorney is not subject to discipline for taking the case because the relevant information he obtained while working as a government attorney is not confidential. The general rule is that a government lawyer who receives confidential government information about a person must not later represent a private client whose interests are adverse to that person, if the information could be used to the material disadvantage of that person. [ABA Model Rule 1.11(c)] The rule covers only "confidential" information, which means information that the government is prohibited from revealing or has a privilege not to reveal, and which is not otherwise available to the public. Here, because the information is available under the Freedom of Information Act, it is not confidential. In fact, any attorney representing the client could obtain the information; thus, the attorney is free to use it. (A) is wrong because an attorney is not barred from ever working on a case where he gained any relevant information while working for the government. To bar representation, the information must be confidential. (B) is wrong because this type of consent is required when the attorney takes on a representation in private practice in a matter in which the lawyer participated personally and substantially while in government service. A "matter" is a set of specific facts involving specific parties. Here, the attorney was not involved in any matter while in government service that concerned the client's claim against the chemical company. (D) is wrong because the information is not confidential and thus can be used against the chemical company. Furthermore, even if the information were confidential, mere nonuse would not be sufficient; the attorney would not be permitted to represent the client.

The driver of a car and his passenger were injured as the result of a collision with a bus. They believe the bus driver was entirely at fault, and they want to bring a negligence action against the bus company. They engage in an initial consultation with a local attorney. In the course of the consultation, the attorney realizes that the bus at issue belongs to a bus company that the attorney's firm is representing in an unrelated matter. The attorney interrupts the conversation, explains this potential conflict of interest, and obtains the written consent of both the driver and passenger to represent them. Which of the following is not true? response - incorrect A The attorney may not represent the car's driver and passenger because he has not obtained the written consent of the bus company. B The attorney may not represent the car's driver and passenger because he did not inform the passenger that he may have a cause of action against the car driver. C The attorney may represent the car's driver and passenger. D The attorney may appear on the bus company's behalf at a court hearing that afternoon.

C. The attorney may not represent both the passenger and the driver when there is a potential conflict of interest between them unless: (i) the attorney reasonably believes that he can represent both clients effectively; and (ii) the passenger and the driver give informed consent, confirmed in writing. The attorney must withdraw from the joint representation, however, if later discovery shows that the passenger has an actual claim against the driver. Here, there is no indication that the driver, the passenger, and the bus company all gave informed consent, confirmed in writing, and thus (C) is not a true statement. [See ABA Model Rule 1.7] (A) is a true statement for the same reason. (B) is true because a lawyer must not represent a client if the representation of that client will be directly adverse to the representation of another client, unless both clients give informed consent, confirmed in writing. [See ABA Model Rule 1.7] (D) is true because the attorney already represents the bus company in the unrelated matter.

A swimming coach was charged with assault of another coach. The swimming coach hired a criminal attorney to defend him. Subsequently, the swimming coach pleaded not guilty and was released on his own recognizance. At his first trial, a jury was empanelled, and the prosecutor was almost finished presenting the testimony of her first witness when a signal from her electronic pager interrupted her. The trial judge granted her request for a short recess, at the end of which the prosecutor told the judge that her office had instructed her not to proceed with this case at this time. The judge responded that if the prosecutor stopped now, the defendant would go free. When the prosecutor indicated that she understood, the judge entered a judgment of acquittal and set the swimming coach free. Twenty days later, the prosecutor recharged the swimming coach with the same offense. The swimming coach hired his original criminal attorney to defend him. The same judge presided over the second trial. The swimming coach's attorney made no pretrial motions. This time the prosecutor did not falter, and in due course the jury at the second trial found the swimming coach guilty as charged. The judge sentenced him to prison for the period required by law, but she stayed the sentence and released him on his own recognizance pending appeal. The swimming coach reluctantly paid the criminal attorney's bill for the second trial—$5,000. However, the swimming coach hired a new lawyer for the appeal, and in due course the appellate court reversed the conviction and set aside the prison sentence. The appellate court's opinion stated it had never seen a clearer double jeopardy violation. Will the swimming coach's original criminal attorney be subject to civil liability in a legal malpractice action brought by the swimming coach for having missed the double jeopardy issue? response - incorrect A No, because the swimming coach never served jail time as a result of the original attorney's error. B No, even if the swimming coach proves by a preponderance of evidence that he did not commit the assault on the opposing coach. C Yes, provided that the swimming coach proves by a preponderance of evidence that he did not commit the assault on the opposing coach. D Yes, but the swimming coach can recover only nominal damages.

C. The criminal attorney will be subject to civil liability in a legal malpractice action brought by the swimming coach for failing to object to the second trial on double jeopardy grounds. A reasonably competent criminal defense attorney would know that a defendant is put in jeopardy when a jury is empanelled and sworn, not to mention that the prosecutor started presenting her case-in-chief. The swimming coach was obviously a proper plaintiff, and he was injured by the criminal attorney's error; he should be able to recover at least part of the $5,000 attorneys' fee, plus damages for his anguish and for the reputational injury caused by the conviction at the second trial. Note that (C) provides that in the malpractice action the swimming coach must prove by a preponderance of the evidence that he was innocent of the underlying criminal offense. That is required by the law of most states that have ruled on the issue. [See Restatement §53, comment d] Observe that in this particular case, a good argument can be made for allowing the swimming coach to recover even without proof of innocence. Here, the malpractice was the criminal attorney's failure to object to the second trial. If the attorney had acted competently, the second trial would never have taken place, and the swimming coach would have lawfully gone free, even if he were unquestionably guilty of the assault. [See Levine v. Kling, 123 F.3d 580 (7th Cir. 1997)—Judge Posner's dictum] (A) and (D) are incorrect because they overlook the $5,000 fee and other less tangible injuries the swimming coach suffered.

Continuously since 1910, the law firm of Alpha & Beta has practiced under that name. The founders of the firm are long dead. No partner named Beta now practices with the firm. Two partners named Alpha were practicing with the firm, but one recently left because she was appointed to the state supreme court. May the firm continue to use the name Alpha & Beta? response - incorrect A No, because no partner named Beta now practices with the firm. B No, because one partner named Alpha left the firm to enter public service. C Yes, unless the firm name would be misleading. D Yes, even if the firm name will mislead some prospective clients.

C. The firm may continue to use the name Alpha & Beta if it is not misleading. ABA Model Rule 7.5(a) permits a firm to practice under a trade name, provided that the trade name is not misleading in violation of ABA Model Rule 7.1. (A) is wrong because a firm may continue using the name of a deceased partner. [Comment 1 to ABA Model Rule 7.5] (B) is wrong because the person who left was not a name partner. Generally, when a name partner enters public service and is not in private practice for a substantial period, the firm must cease using that person's name. [ABA Model Rule 7.5(c)] Here, however, the Alpha surname refers to the deceased founder. If, however, the use of the Alpha name in the firm name would mislead potential clients (e.g., by making them think they could gain an advantage in the state supreme court by hiring that firm), then continued use of the name would violate the ethics rules. [See ABA Model Rule 7.1] (D) is wrong because the rules on firm names are subject to the more general provisions on misleading communications.

The state bar association has established a peer counseling program whereby lawyers who are addicted to alcohol or other drugs can receive confidential counseling from other lawyers. The bar association's ethics rule on confidential information provides that communications between the counselor lawyer and the counseled lawyer are to be treated just like confidential communications between an attorney and client. A lawyer is addicted to alcohol and is receiving peer counseling under the program from another lawyer. The lawyer is a large, strong man, and his addiction has made him subject to periodic fits of physical violence. This afternoon, during their peer counseling session, the lawyer told his peer counselor that his client had refused to pay the fees he owes, and that he intended to punch out the client the next time he got roaring drunk. From working with the lawyer over an extended period, the peer counselor believes that he may really do it. May the peer counselor disclose the lawyer's statement to the client and the police? A No, unless the lawyer consents. B No, unless the peer counselor is certain that the lawyer will carry out his threat. C Yes, even if the lawyer objects. D Yes, because he is serving as a peer counselor, not a lawyer.

C. The peer counselor may disclose the statement even if the lawyer objects. The state ethics rule on confidentiality treats communications between a lawyer and his peer counselor just like communications between an attorney and a client. If the counselor had heard one of her clients make this threat, she could have warned the police and the intended victim. An attorney may reveal confidential information to the extent she reasonably believes necessary to prevent reasonably certain death or substantial bodily harm. [ABA Model Rule 1.6(b)(1)] Thus, the peer counselor may warn the client and the police. (A) is wrong because the peer counselor may act to prevent the lawyer from causing substantial bodily harm. The lawyer's consent is not necessary. [ABA Model Rule 1.6(b)(1)] (B) is wrong because if an attorney reasonably believes that her client (or anyone else) is about to inflict substantial bodily harm on someone, she may take steps to prevent it, even if she is not certain that the client (or other person) will do it. (D) is wrong because the state ethics rule on confidentiality equates the peer counselor relationship with the relationship between an attorney and client; thus, the ability to disclose is the sam

A family-owned chemical company receives information that some of its sales associates may have entered into price fixing agreements in violation of federal antitrust laws. The shares of the chemical company are not publicly traded and the company is not subject to the jurisdiction of the Securities and Exchange Commission. The general counsel of the corporation hires an outside antitrust lawyer to investigate the matter. The antitrust lawyer and her associate investigated the matter and discovered that some of the sales associates had indeed entered into agreements that could make the corporation civilly and criminally liable under the antitrust laws. The antitrust lawyer reported these findings and her antitrust advice in a confidential letter addressed jointly to the company's chief executive officer and the general counsel. The general counsel wrote back, asking the antitrust lawyer to stand by to defend the company if needed. Months went by, and the antitrust lawyer heard nothing more. Her associate grew restless, and without telling the antitrust lawyer, he told a friend in the Justice Department what the company's salespeople had done. The Justice Department began a price fixing investigation of the company and its competitors. Which of the following is correct? response - incorrect A The antitrust lawyer is subject to discipline for failing to report the company's situation to the antitrust enforcement authorities in the Justice Department, but her associate's conduct was proper. B The conduct of both the antitrust lawyer and the associate was proper, and neither of them will be subject to civil liability if the company sues them for legal malpractice. C The associate is subject to discipline for tipping off the Justice Department, but the antitrust lawyer's conduct was proper. D Neither the antitrust lawyer nor the associate is subject to discipline, but both of them may be subject to civil liability if the company sues them for legal malpractice.

C. This question is governed by ABA Model Rule 1.13, not by the SEC's regulations under the Sarbanes-Oxley Act, because the company is not publicly owned and is not subject to the jurisdiction of the SEC. The associate is subject to discipline for tipping off the Justice Department because he violated the duty of confidentiality imposed by ABA Model Rule 1.6(a). When the antitrust lawyer and the associate investigated the price fixing rumor at the request of the company's general counsel, they were operating under ABA Model Rule 1.13(d), which applies to lawyers who are hired "to investigate an alleged violation of law" or to "defend an organization" or its people against a claim arising out of an alleged violation of law. That means that ABA Model Rule 1.13(c) does not apply, and they must not report to outsiders about what they find. Both parts of (A) are wrong—the antitrust lawyer acted properly, and the associate is subject to discipline, as explained above. (B) is wrong because the associate's conduct was not proper. Moreover, the associate probably committed legal malpractice when he tipped off the Justice Department, and the antitrust lawyer might be vicariously liable for his malpractice because she was his supervisor. (D) is wrong because the associate is subject to discipline for violating ABA Model Rule 1.6(a).

A law student is applying to become a member of the state bar. The bar application questionnaire asks whether the applicant has ever used any illegal drug in violation of state law. When the law student was in high school, she occasionally smoked marijuana, which is a misdemeanor under state law. However, the statute of limitations has run on these incidents, and the law student believes that she could not validly be kept out of the state bar for those offenses. She therefore believes that the question is irrelevant and an invasion of her privacy in violation of the state constitution. She fears, however, that challenging the question could brand her as a troublemaker and delay her admission to the bar. Which of the following would be proper? response - incorrect A Answer the question in the negative, without saying more. B Answer the question in the negative, citing the state constitution's privacy provision. C Decline to answer the question, citing the state constitution's privacy provision. D Decline to answer the question, citing the federal constitutional privilege against self-incrimination.

C. While a bar applicant must cooperate in reasonable investigations by the state bar and make disclosures relevant to her fitness to practice, she may challenge the validity of a question on legally tenable grounds. [See ABA Model Rule 3.4(c)] That a question violates the state constitution would be a legally tenable argument. Moreover, by declining to answer, the law student has not made an untrue statement. (A) and (B) would not be proper because a bar applicant must not make untrue statements on a bar application. [ABA Model Rule 8.1(a)] If the law student answers in the negative, she will have made an untrue statement. (D) would not be proper because the self-incrimination privilege is inapplicable here. Even if the privilege against self-incrimination applies to questions on bar applications (a debatable proposition), it does not apply when criminal punishment is barred by the statute of limitations. [See Hazard & Hodes, §62.6]

A federal prosecutor is stationed in State A and is gathering evidence to support federal racketeering charges against a swindler. The prosecutor believes in good faith that a wealthy banker in State B has personal knowledge about three federal felonies committed by the swindler, but the banker will not disclose what he knows. The prosecutor discovers from a secret informant that the banker illegally drained off $4.7 million from a failing State B bank—a state felony punishable by 10 years in prison. The prosecutor therefore mails a letter from his office to the banker, stating in relevant part: "I am coming to State B next week. If you don't give me what I need concerning the swindler, I am going to tell the State B prosecutors what you did to that bank." Is the federal prosecutor subject to criminal liability because of his evidence-gathering technique? A No, because the federal prosecutor had a legal right to tell the State B prosecutors about the banker, and he simply warned the banker what he intended to do if the banker did not cooperate. B No, because the federal prosecutor was acting in good faith, believing that the banker had relevant, unprivileged information that was material to an ongoing federal criminal investigation. C Yes, because the federal prosecutor made an interstate threat to accuse the banker of a crime for the purpose of extracting valuable information that he could use against the swindler. D Yes, because the federal prosecutor acted under color of law to deprive the banker of his federally protected civil rights.

C. he federal prosecutor is subject to criminal liability because his evidence-gathering technique amounted to extortion, an interstate threat against the banker for the purpose of extracting information to use against the swindler. Under modern statutory law, the crime of extortion covers obtaining anything of value, tangible or intangible, by making various kinds of threats, including a threat to accuse a person of a crime. [See, e.g., 18 U.S.C. §875(d)—felony to obtain something of value by transmitting in interstate commerce a threat to accuse a person of a crime; see also Perkins & Boyce, 442-52] The information that the federal prosecutor wanted from the banker was something of value in the swindler investigation, and the prosecutor clearly threatened to accuse the banker of a crime if the banker did not cooperate. Furthermore, the threat was transmitted interstate, thus bringing it within the scope of the federal statute cited above. (A) is incorrect because the federal prosecutor did have a legal right to tell the State B prosecutors about the banker and the savings and loan, but the federal prosecutor did not have a right to threaten to do so (i.e., to commit extortion) in order to coerce the banker to cooperate. (B) is incorrect because it ignores the law of extortion. Just as it would have been illegal for the federal prosecutor to extract the information by physical torture, it was illegal to extract it by extortion. (D) is not as good as (C) because it is vague and because the right to be free from extortion, though important, is not a federally protected civil right.

A lawyer is interested in obtaining legal business from a mineworkers' union that has many members in the state in which the lawyer practices. As a result of a recent mine fire and explosion in which several union members were killed, the union has succeeded in persuading the appropriate state agency to bring an administrative action against the company that owns the mine for failing to install smoke detectors, which might have saved some lives in the disaster. Although the lawyer is in no way involved in the case, he sees this as an opportunity to obtain future business from the union by showing the union that he is strongly on its side in the mine disaster case. The lawyer telephones a popular call-in radio show, and says that he is shocked and appalled at the callousness of the mining company that caused the recent disaster in which so many miners were killed. He further opines that the mining company was willful and wanton in its failure to install smoke detectors, and expresses hope that the company will not be allowed to escape the consequences of its despicable conduct. Without the lawyer's knowledge or consent, his statement was later printed in several news-papers in the state. Is the lawyer subject to discipline for his conduct? A Yes, because he was substantially motivated by his desire to attract fee-paying business. B Yes, because lawyers must not make public comments concerning pending litigation. C No, because he did not make any false or misleading claims about himself or his services. D No, because the statement was printed in the newspapers without his knowledge or consent.

C. The lawyer is not subject to discipline because he did not make any false or misleading claims about himself or his services. Lawyers, like other citizens, have the right to express their views in the media on newsworthy issues. Even if a lawyer's sole purpose in seeking media publicity is to lure clients, the state may not impose professional discipline on the lawyer absent a compelling state interest. A lawyer who uses the media to lure clients may, however, be disciplined for making statements or claims that are false or misleading about the lawyer or his services. [ABA Model Rule 7.1] Here, the lawyer made no statements about himself or his services other than the fact that he is a lawyer and his opinion about the incident. There is nothing false or misleading in his communication. (A) is wrong because, as discussed above, the fact that the lawyer was motivated by the desire to attract fee-paying business is irrelevant. This is not a case of in-person or live electronic solicitation, which is the only circumstance in which this consideration is relevant. (B) is wrong because it is overbroad. Lawyers can and do make public statements about pending litigation all the time. Lawyers who are involved in a proceeding cannot make statements that they know will have a substantial likelihood of materially prejudicing an adjudicative proceeding. [ABA Model Rule 3.6(a)] That is not the case here; the lawyer has no reason to believe his opinion will materially prejudice the state agency bringing the action. (D) is wrong because the fact that the statement was printed in the newspapers is of absolutely no consequence; it does not affect the propriety of the lawyer's behavior regardless of whether he gave consent.

A lawyer in a partnership died, leaving his daughter, a doctor, as his sole heir. Under the partnership agreement, the firm plans to make the following payments to the deceased lawyer's daughter: $100,000, which represents the decedent's share of the firm's assets, as measured by his capital contribution; $45,000, which represents the decedent's share of fees that had been earned but not collected from clients at his death; and a $125,000 death benefit, representing a percentage of the decedent's earnings the year prior to his death, and payable in 12 monthly installments. Under the Model Rules, which of the following represents the most that the firm may properly pay to the decedent's daughter? A $100,000 for the decedent's share of the firm's assets. B $145,000 for the decedent's share of the firm's assets and his share of uncollected fees. C $170,000, which represents the death benefit and the decedent's share of uncollected fees. D $270,000, which includes the decedent's share of the firm's assets, his share of uncollected fees, and the death benefit.

D The firm may pay all of the money as planned. Even though the decedent's daughter is a nonlawyer, the firm may make certain kinds of payments to her from money originally earned as legal fees. [ABA Model Rule 5.4(a)(1)] The $100,000 is a proper payment because it reflects the decedent's share of the capital assets of the firm. The $45,000 is a proper payment because the fees it represents had been earned, albeit not collected, at the time the decedent died. The $125,000 is a proper payment because it is a reasonably computed death benefit payable over a reasonable period of time.

A lawyer was a widely admired, highly compensated trial attorney in solo practice. He represented clients in all types of civil and criminal litigation, mostly in high-profile cases that drew a lot of media attention. The governor of the state where the lawyer practiced had been harshly criticized for appointing appellate judges who lacked significant experience as trial counsel. Hoping to silence his critics, the governor appointed the lawyer to serve out the remaining seven years of a recently deceased supreme court justice's 12-year term. After the seven years, the lawyer can run for election to a new 12-year term. Before taking the oath as judge, the lawyer sold his entire law practice—books, client files, office lease, furniture, and goodwill—to another attorney. The lawyer gave appropriate advance notice to the clients, and the purchasing attorney covenanted that he would not raise their legal fees. A few years later, one of the cases that the lawyer transferred to the purchasing attorney came before the state supreme court on appeal. Which of the following propositions is false? response - correct A The lawyer's sale of his law practice was proper. B The purchaser's covenant not to increase the fees paid by the lawyer's clients was proper. C The lawyer must disqualify himself from the case involving his former client. D The lawyer may participate in the decision of the case involving his former client, provided that all of the other supreme court justices give their informed consent.

D. (D) is false. CJC Rule 2.11(C) explains remittal of a judge's disqualification, and remittal requires the agreement of all of the parties and their lawyers, not the other justices. (A) is true. ABA Model Rule 1.17 permits the sale of an entire law practice, or an area of practice, subject to some conditions, all of which are met here. (B) is true. ABA Model Rule 1.17(d) provides that when a practice is sold, the fees charged to the clients cannot be increased by reason of the sale. (C) is true. CJC Rule 2.11(A)(6)(a) says that a judge must disqualify himself if he previously served as a lawyer in the matter.

un for judge. She was successful in her efforts and was duly sworn in as one of the 15 sitting judges on the district court. According to state venue rules, at least 90% of the cases that the attorney usually handles must be filed with the district court. When opposing counsel in one of the attorney's cases learned that the attorney's sister was a judge, he objected. The attorney responded that it was his sister's duty to uphold the law and no one would receive special favors. Is it proper for the attorney to handle such cases and appear before the district court? A No, because an opposing party objected. B No, because it creates an appearance of impropriety. C No, because an attorney should not appear in a district court where a close relative serves as one of the judges. D Yes, because the attorney did not suggest that his clients will receive unfair advantages because his sister is a judge.

D. A lawyer must not imply that he can improperly influence a government official. [ABA Model Rule 8.4] Here, the attorney is not actually appearing before his sister, and so (B) and (C) are incorrect. Without more, the mere objection of an opposing party should not keep the attorney from appearing in district court; thus, (A) is incorrect. (D) is therefore the best answer.

A lawyer was employed to represent a man at his trial for treason; the man was charged with smuggling top secret military information to a foreign government. The lawyer had reason to suspect that some of the prosecution's witnesses were paid liars. Therefore, with his client's consent, the lawyer hired to assist him in the defense a psychologist whose specialty is the behavior of liars. The client consented to the lawyer's advancing the psychologist's fee as a part of the expenses of litigation. At the trial, the psychologist sat with the lawyer at the counsel table. She watched the witnesses testify, and she advised the lawyer when she believed that a witness was lying and ought to be pursued on cross-examination. Most of the time, the lawyer followed the psychologist's advice, but sometimes he did not. Ultimately, the client was convicted and sent to prison for 20 years. Which of the following propositions is correct? response - incorrect A The lawyer is subject to discipline for allowing a third party to interject herself into the relationship between him and his client. B The lawyer is subject to discipline for advancing the psychologist's fee as a part of the expenses of litigation. C The lawyer is subject to discipline for allowing a nonlawyer to sit at the counsel table and to participate in his legal representation of a criminal defendant. D The lawyer is not subject to discipline because he did not give up his discretion to the psychologist, as shown by the fact that he did not always follow her advice.

D. A) reflects a misconception of ABA Model Rule 1.8(f), which prohibits a lawyer from allowing the interests of a third party to interfere with the relationship between the lawyer and the client. That is not the case where an alleged expert like the psychologist is hired to assist the lawyer in representing the client. (C) reflects a similar misconception. Attorneys frequently use a variety of experts to advise them during trial. The lawyer did not give up his discretion to the psychologist, as evidenced by the fact that he sometimes did not follow her advice. Furthermore, such experts frequently sit at the counsel table so they can be close to the lawyer during trial. (B) is incorrect because a lawyer is allowed to advance litigation expenses. [See ABA Model Rule 1.8(e)(1)] Although some lawyers might question the wisdom of spending money on a purported expert on liars, that would seem to be a matter best left to the discretion of the individual lawyer, and the Model Rules do not suggest otherwise. Thus, (D) is correct, and (A), (B), and (C) are incorrect.

An attorney attended a local law school under an honors program, whereby accepted students who had an undergraduate grade point average of 3.8 or higher receive a full tuition-paid scholarship. While in law school, the attorney completed the requirements for a taxation concentration, for which she received a certificate at graduation. She passed her state's bar examination, and she currently limits her practice to tax law. She is drafting an advertisement for her legal services. Her current draft states that: she is an honors graduate of the local law school; she is a certified tax specialist; she practices only tax law; and she likely can save clients money on their federal and state income taxes. The current draft of the advertisement is improper for which of the following reasons? response - incorrect A An attorney may specialize, but may not claim to be a certified specialist in a particular field of law. B The ad is undignified and may cause readers to lose respect for the legal profession. C The reference to tax savings is likely to create unjustified expectations about the results the attorney can achieve. D The ad contains false and misleading information.

D. ABA Model Rules 7.1 and 7.2 provide that communications about legal services must not be false or misleading. Attending a law school under an "honors program" is not the same as being an "honors graduate" of the law school; one could receive high grades as a college undergraduate but graduate at the bottom of her law school class. In addition, a law school concentration certificate is not the same as being a "certified specialist." Accordingly, the attorney's current draft does not comport with ABA Model Rules 7.1 and 7.2. Thus, (D) is correct. (A) is incorrect because an attorney may claim to be a certified specialist under the circumstances specified in ABA Model Rule 7.4(d). (B) is incorrect because the facts do not suggest that the ad was undignified, and the greater problem with the ad is the reason stated in (D). (C) is incorrect due to the modifier "likely" and the lack of specificity regarding the amount of the potential tax savings. [See Comment 3 to ABA Model Rule 7.1—"The inclusion of . . . qualifying language may preclude a finding that a statement is likely to create unjustified expectations or otherwise mislead the public."]

Three months from now, a local judge will be up for reelection. His opponent is a bright and ambitious young attorney in the prosecutor's office. Due to the press of his judicial duties, the judge has not paid much attention to the upcoming election. One afternoon an old law school friend visits him in his chambers. She convinces him that he must get busy if he hopes to defeat his opponent. The friend frequently represents clients in the judge's court, and he trusts her judgment and ability; thus, he agrees to have her serve as chairperson of his reelection campaign committee. In her capacity as the chairperson of the judge's reelection campaign committee, the friend contributed $200 of her own money to the campaign fund. She also urged other lawyers who appeared in the judge's court to make contributions to his campaign fund, and further urged those lawyers to allow their names to be listed in a half-page newspaper advertisement in favor of the judge's reelection. Finally, the friend arranged for the taping of a television advertisement in which the judge pledged never to believe the sworn testimony of a criminal defendant when there was conflicting sworn testimony from a law enforcement officer. Which of the following was improper in the context of the judge's campaign for reelection? response - incorrect A Contributing $200 to the judge's campaign fund. B Urging other lawyers who appear in the judge's court to make contributions to the judge's campaign fund. C Urging other lawyers who appear in the judge's court to allow their names to be listed in a half-page newspaper advertisement in favor of the judge's reelection. D Creating the television advertisement.

D. CJC Rule 4.4 allows a candidate for an elected judicial office to have a campaign committee to solicit campaign funds and publicly stated support for the candidate. No provision of either the CJC or the ABA Model Rules prohibits a lawyer who frequently appears before a judge from contributing to or publicly endorsing the campaign. Thus, (A), (B), and (C) are all proper here. However, (D) was improper because judges and judicial candidates are not permitted to "make pledges, promises or commitments that are inconsistent with the impartial performance of the adjudicative duties of judicial office." [CJC Rule 4.1(A)(13)]

A solo practitioner is one of only three lawyers in a small town. The solo practitioner is presently defending a client in a criminal action for assault and battery. This morning one of the solo practitioner's regular clients, a gas and grocery store, asked the solo practitioner to sue the same client to recover a past due amount on a gasoline and grocery charge account. Would it be proper for the solo practitioner to represent the gas and grocery store in the charge account case? A No, because it is presumed that a lawyer obtains confidential information in the course of representing a client. B No, unless the other two lawyers in town are disqualified from representing the gas and grocery store. C Yes, because there is no substantial relationship between the charge account case and the assault and battery case. D Yes, if both the client and the gas and grocery store consent after full disclosure of the conflict, and such consent is confirmed in writing.

D. It would not be proper for the solo practitioner to represent the grocery store unless both the client and the grocery store give informed consent, confirmed in writing, to the representation. A lawyer who is presently representing a client in one litigation matter should not simultaneously oppose that client in a different litigation matter, even if the two matters are unrelated. [ABA Model Rule 1.7(a) and comment 6] The purpose of the rule is to avoid putting the client into the difficult position of treating the lawyer simultaneously as friend and foe. The conflict can be solved only by informed consent, confirmed in writing, from both the client and the grocery store. (A) is wrong because the rule prohibiting the representation applies regardless of whether the solo practitioner has obtained relevant confidential information. (B) is wrong because the conflict is not obviated by the shortage of legal talent in the small town. (C) is wrong because the Rule applies even if the two cases are unrelated.

A bank operates a professional referral hotline for its depositors. Any bank depositor who needs to find a physician, lawyer, accountant, dentist, or the like can telephone the hotline and obtain a free referral from lists of professionals compiled by the bank. The lists are limited to professionals who maintain an average balance of at least $10,000 in an account at the bank, but the professional does not pay a fee to the bank for receiving a particular referral. A lawyer keeps $10,000 on deposit with the bank for the express purpose of being included on its lawyer referral list. Is this arrangement proper? A Yes, because the bank is functioning in the role of a lawyer referral service. B Yes, because neither the bank's depositors nor the professionals pay a fee for referrals. C No, because the lawyer is required to keep $10,000 on deposit to be included on the list. D No, because this arrangement constitutes an association with a nonlawyer for the practice of law.

D. The arrangement is not proper because the lawyer is required to keep $10,000 on deposit to be included on the list. A lawyer may not give "anything of value" to a person for recommending the lawyer's services. [ABA Model Rule 7.2(b)] The bank benefits in many ways by increasing the amount of its deposits; for example, its deposits determine how much it can lend to borrowers. Thus, obtaining deposits from lawyers is of value to the bank, and that is one reason it has devised the referral scheme. (A) is wrong because although a lawyer may pay the usual charges of a not-for-profit or qualified lawyer referral service [ABA Model Rule 7.2(b)], banks operate for profit, and there is no indication that the bank has been approved by the appropriate regulatory authority as a qualified lawyer referral service.

A young associate was assisting a senior partner in writing the reply brief in an appeal for one of the partner's clients. In doing the legal research, the associate discovered a recent case from the controlling jurisdiction that had not been cited in the adversary's brief. In the associate's opinion, the case was directly opposed to the position of the partner's client. The associate asked the partner about citing it in the reply brief, but the partner explained that, in his view, the case was not directly on point and did not have to be cited. The associate and the partner argued back and forth at some length and finally decided to submit the question to one of the other senior partners in the firm for a fresh view. That partner sided with the other partner, and the reply brief was filed without mentioning the case. May the associate write a short letter to the appellate court and the adversary lawyer, explaining his position and enclosing a copy of the case? response - correct A Yes, because the associate had a duty to call the case to the court's attention. B Yes, because the associate must not allow another person to interfere with his professional judgment. C No, because the associate must not communicate with a court ex parte about the merits of a pending case. D No, because the associate should abide by the partner's resolution of the matter.

D. The associate should abide by the partner's resolution of the matter. A subordinate lawyer does not violate the Rules of Professional Conduct by acting in accordance with a supervisor's reasonable resolution of an arguable question of professional duty. [ABA Model Rule 5.2(b)] Here, it seems clear that the question was arguable because the third attorney called in to determine the relevance of the case also felt it was not on point. (A) is wrong because the associate only has a duty to call the case to the court's attention if the case is directly on point. That is a debatable question, and the associate's supervisors have determined the case is not directly on point. Thus, the associate need not reveal the case. (B) is wrong because this is not the situation intended to be addressed by the rule against allowing a third party to influence the lawyer's judgment, which usually arises when a third party pays the lawyer's fees to represent another. Of course, a subordinate lawyer should be influenced by his supervisor. That is not an excuse for clearly unethical conduct, but on a debatable issue, such as the one presented here, the subordinate lawyer is free to defer to the supervisor's judgment. (C) is wrong because in most jurisdictions a lawyer may communicate in writing with the court about the merits of a pending case if he sends a copy to opposing counsel. This communication is not considered ex parte. [See Restatement §113, comment c]

An attorney has organized his law practice as a professional corporation. The attorney is the sole shareholder. The sign on the office door states: "Professional Corporation— Attorney at Law Corporate and Business Law, Torts and Domestic Relations" The attorney has one lawyer-employee, who was admitted to practice two years ago. The attorney pays his employee a modest monthly salary plus 60% of the fees collected in cases that the employee handles by herself. The attorney has a general business practice and is not a certified specialist in any practice area. When a client needs representation in a tort or domestic relations matter, the attorney turns the case over to his lawyer-employee. When the attorney turns a case over to the employee, he provides general guidance and is available to answer any questions she may have, but he does not supervise every step she takes. Is the attorney subject to discipline? response - incorrect A Yes, because he splits fees with his employee in matters she handles by herself. B Yes, because he does not closely supervise the work done by his employee. C No, but he should change his sign to show the fields of practice that he personally handles. D No, because the employee is a lawyer-employee of the attorney.

D. The attorney is not subject to discipline. Because the employee is a lawyer-employee of the attorney, she is regarded as being "in the same firm." She and the attorney are thus allowed to split fees without complying with the rules that govern fee splits between lawyers who are not in the same firm. [ABA Model Rule 1.5(e)] (A) is wrong for the reason just stated. (B) is wrong because no Rule requires the attorney to supervise the lawyer-employee at every turn, so long as he takes reasonable steps to assure that she performs her work competently and otherwise within the bounds of legal ethics. [See ABA Model Rule 5.1—supervisory duties of lawyers within a firm] (C) is wrong; a law firm may state particular fields of law that the firm's attorneys handle. [ABA Model Rule 7.4(a)]

Two years ago, an attorney represented his client when he sold his property. Unbeknownst to the attorney, the client made some fraudulent statements to the buyer about the value of some mineral deposits on the property. The buyer recently discovered the fraud and is now in the attorney's office threatening to immediately file a civil fraud suit against both the client and the attorney. The buyer accuses the attorney of engineering the fraud and helping his client carry it out. The only way that the attorney can convince the buyer that he had no part in the fraud is to tell the buyer a fact that the client disclosed to him in the deepest confidence when he was working on the property transaction. May the attorney disclose the fact without the consent of the client? response - incorrect A No, if doing so will harm the client. B No, because doing so would breach his duty of confidentiality to the client. C Yes, but only after the buyer files the civil fraud suit against him. D Yes, even if doing so will subject his client to civil or criminal liability.

D. The attorney may reveal the confidence even if doing so will subject his client to civil or criminal liability. A lawyer may disclose a client's confidence "to establish a defense to a criminal charge or civil claim against the lawyer based upon conduct in which the client was involved . . . ." [ABA Model Rule 1.6(b)(5)] Although the lawyer must wait until the assertion of misconduct arises, he need not await the filing of a formal charge or complaint. The lawyer may defend himself by responding directly to a third party who has made such an assertion. [See Restatement §64, comment c] (A) is wrong because the lawyer may disclose the fact even if doing so harms the client. (C) is wrong because the lawyer need not wait for the complaint to be filed, as explained above. (B) is wrong because it ignores the self-protection exception to the general rule of confidentiality.

A plaintiff brought a civil action to recover damages for personal injuries he suffered as the victim of alleged police brutality inflicted by three defendant police officers. The trial was widely reported by the media. The jury returned a verdict in favor of the plaintiff and against the three police officers for $500 million. When the trial judge received the verdict, he was shocked by the size of the award. Before dismissing the jurors, the judge told the jurors that when they were sworn in, they had promised that they would deliver a verdict based on the evidence and that they would not be swayed by passion or prejudice. The judge further admonished the jurors that they had failed in those duties, that they had made a mockery of justice, and that they should be ashamed of themselves. He then dismissed the jury, and the defense lawyers renewed their motion for judgment as a matter of law and, alternatively, moved for a new trial. The judge announced that he would rule on the motions the following Monday at 10 a.m. in open court. The press reports of the verdict and the judge's comments to the jury created a great public tumult in the city where the case was tried. On the following Monday, the courtroom was jammed with reporters. Primarily for the purpose of educating the reporters, the judge first gave a detailed explanation of the legal requirements for granting a renewed motion for judgment as a matter of law and for granting a new trial motion. He then granted the renewed motion for judgment as a matter of law and, alternatively, the motion for a new trial. Were the judge's actions proper? A Both the statements to the jury and the communication with the reporters were proper. B Neither the communication with the reporters nor the statements to the jury were proper. C The statements to the jury were proper, but the communication with the reporters was not. D The communication with the reporters was proper, but the statements to the jury were not.

D. The communication with the reporters was proper, but the statements made to the jury were not. A judge should not "commend or criticize jurors for their verdict other than in a court order or opinion in a proceeding." A judge's commendation or criticism may impair a juror's ability to be impartial in a subsequent case. [CJC Rule 2.8(C)] The judge's statements to the jury clearly violate this Rule; thus, (A) and (C) are incorrect. Although judges should not comment on pending cases in a manner that might interfere with fairness, judges are not prohibited from making public statements in the course of their official duties or from explaining for public information the procedures of the court. [CJC Rule 2.10(D)] The judge's explanation of the motions to the reporters was an appropriate way to inform the public of the meaning and significance of the judge's decision to reject the $500 million verdict; thus, (B) and (C) are incorrect.

An attorney is admitted to practice only in State A, where he specializes in securities and real estate finance law. In that role, the attorney advised his client that the law of State B did not require the client to include information about certain mineral rights in a disclosure statement that the client had to file in State B in order to sell some real estate limited partnership interests to State B citizens. Acting on the attorney's advice, the client did not disclose the information and did sell partnership interests to State B citizens. Later, the attorney became a full-time trial court judge in State A. Later still, State B brought a criminal action against the client for failing to disclose the mineral rights information in his State B disclosure statement. One of the client's defenses is that he lacked the necessary criminal intent because he was acting in good faith based on the advice of his counsel, the attorney who is now a judge. The client needs the judge's testimony to prove that the judge did indeed advise him that he was not required to disclose the mineral rights information. The judge, in State A, is beyond the subpoena power of the State B court. May the judge voluntarily testify on behalf of the client? A No, because judges are disqualified from serving as witnesses in criminal cases. B No, because he is not admitted to practice in State B, and his testimony about State B law would be inadmissible. C Yes, because a judge may testify as a witness, except in his own court or one under its appellate jurisdiction. D Yes, because his testimony would concern the giving of the advice, not his client's character.

D. The judge may testify because he is testifying to facts, not the defendant's character. CJC Rule 3.3 prohibits a judge from testifying voluntarily as a character witness, but it says nothing about serving as an ordinary fact witness. (A) is wrong because there is no such rule. Judges are not disqualified from testifying in criminal cases. (B) is wrong because an attorney need not be admitted to practice in State B in order to advise a client about State B law. Even if that were untrue, the judge's testimony would still be admissible as evidence of the client's lack of criminal intent. (C) is wrong because it states a nonexistent rule. While a judge is not competent to be a witness at a trial over which he himself is presiding [see Fed. R. Evid. 605], no rule forbids a judge from serving as an ordinary fact witness in a case that is pending before a different judge in his own court or a court that is under his court's appellate jurisdiction.

An inventor asked a patent lawyer to represent him in obtaining a patent on a new computer technique for predicting the growth patterns of tumors in the human body. The lawyer informed the inventor that he had never worked on that kind of patent application before, and that he would have to do extensive background research on the patentability of computer techniques. The lawyer will be able to use the knowledge that he gains through the research to serve other clients who wish to obtain patents for all manner of other computer techniques. The lawyer offered to do the work for the inventor for his standard hourly rate, but the inventor proposed instead to assign the lawyer a 10% interest in the patent, if and when it was issued. The lawyer agreed to do the work on that basis, and he and the inventor entered into an appropriate written fee agreement. The lawyer did the work; the patent was ultimately issued and proved so valuable that the lawyer was able to sell his 10% interest for $9.7 million (which was a reasonable fee for the work performed). Is the lawyer subject to discipline? response - incorrect A Yes, because he acquired a proprietary interest in the subject of the representation. B Yes, because it is unreasonable to charge one client for background research that will be used to earn fees from other clients. C No, because the inventor agreed to the fee arrangement. D No, because $9.7 million is not an unreasonably high fee for the work that the lawyer did.

D. The lawyer is not subject to discipline because $9.7 million is within the bounds of reason as a fee for the work the lawyer did. [See ABA Model Rule 1.5(a)] Among the various factors that point to the reasonableness of the lawyer's fee are: the novelty and difficulty of the patentability issue, the fact that the inventor was the one who suggested the fee arrangement after having been offered a standard hourly fee, the value of the result that the lawyer obtained for the inventor, and the contingent nature of the arrangement, which imposed a high risk on the lawyer. (A) is wrong because a lawyer is prohibited from acquiring a proprietary interest in the subject of litigation he is conducting [ABA Model Rule 1.8(i)], and obtaining a patent is not litigation. Even if this were a litigation case, the contingent fee exception to the rule would apply. [ABA Model Rule 1.8(i)(2)] (B) is wrong because a fee that is otherwise reasonable does not become unreasonable simply because the lawyer can use the knowledge gained to earn fees from other clients. (C) is wrong because the mere fact that the client agreed to the fee arrangement does not by itself make the fee reasonable. Many factors, including the time, labor, and skill required to do the job, are considered in determining whether the fee is reasonable. [ABA Model Rule 1.5(a)]

An author wrote a best-selling novel based on the life and crimes of John Dillinger, the famous bank robber. The author sold the movie rights to a film producer, who promised to pay the author a lump-sum royalty of $5 million upon the release of the movie. After the producer hired an actor to play the lead role and made other expensive preparations for filming, the author repudiated the contract. The producer hired a lawyer to sue the author for a declaratory judgment that the contract was valid and enforceable. At the producer's request, the lawyer agreed to do the legal work on a contingent fee basis: If the producer wins, the lawyer will be paid 1.75% of the gross receipts from the movie, but if the producer loses, the lawyer will be paid nothing. The producer and the lawyer entered into a written fee agreement that contains all the details required by the rules of legal ethics. Which of the following statements is true? A The lawyer is subject to discipline for entering into a publication rights contract with his client. B The lawyer is subject to discipline for acquiring a personal interest in the subject of the litigation. C The lawyer's fee agreement is proper, but only if the author gives informed consent. D The lawyer's fee agreement is proper, even though it gives the lawyer a personal interest in the subject of the litigation.

D. The lawyer's fee agreement is proper even though it gives the lawyer a personal interest in the subject of the litigation. The lawyer has acquired a personal interest in the movie, which is in one sense the real subject of the litigation. However, the rule against acquiring a personal interest in the subject of litigation has an exception that allows a lawyer to represent a client for a contingent fee. [ABA Model Rule 1.8(i)(2)] (A) is wrong because the rule on literary rights contracts covers only literary works based in substantial part on information relating to the representation. Here, the movie concerns John Dillinger, not the producer. [See ABA Model Rule 1.8(d)] (B) is wrong because, as discussed above, a lawyer may acquire an interest in the subject matter of the litigation in the form of a contingent fee. (C) is wrong because the fee agreement between the producer and the lawyer does not require the author's informed consent. The author is not a current or former client of the lawyer, and the author has no apparent interest that would cause the lawyer a conflict and force him to disclose and explain the material risks and available alternatives and obtain the author's consent.

A public defender was assigned to represent a defendant at the defendant's preliminary hearing on a charge of kidnapping for ransom. Against the public defender's advice, the defendant testified on his own behalf at the preliminary hearing. The defendant was bound over for trial. At that point, the defendant's brother provided money to hire a private lawyer to represent the defendant, and the public defender was discharged. The defendant testified on his own behalf at the trial, and the matter concluded after the jury acquitted him. Later, in connection with his work on another matter, the public defender read the transcript of the defendant's trial. Based on information the public defender learned while representing the defendant, the public defender concluded that the defendant had committed perjury, both at the preliminary hearing and at the trial. However, the public defender does not believe that the defendant poses a danger to the community. May the public defender reveal the defendant's perjury? A Yes, the public defender may reveal the perjury committed at the defendant's preliminary hearing. B Yes, the public defender may reveal both instances of perjury. C No, because the public defender does not believe that the defendant poses a danger to the community. D No, because disclosure would violate the public defender's duty of confidentiality.

D. The public defender may not reveal the defendant's perjury because to do so would violate the public defender's duty of confidentiality. No exceptions to the confidentiality requirement apply to these facts. There is no indication that revealing the perjury is necessary to prevent reasonably certain death or substantial bodily harm. Also, there is no indication that the defendant's perjury is a crime that is reasonably certain to result in substantial injury to the financial interests of another, in furtherance of which the defendant has used the public defender's services. The obligation to reveal perjury under the Model Rules does not apply because that obligation ceases at the end of the proceedings, and both proceedings here have concluded. (A) is wrong because the proceeding has ended and, therefore, the public defender is obligated to keep the information in confidence. (B) is wrong for the same reason. Furthermore, even if the trial was ongoing, the public defender would not be obligated to reveal the perjury because he was not representing the defendant at trial. The public defender represented the defendant in the preliminary hearing, which has concluded, ending his obligation to disclose. (C) is wrong because what the public defender believes about the defendant's dangerousness is not relevant. There is no exception to the duty of confidentiality based on the client's violent propensities.

A steel company merged with an iron corporation. The state attorney general sued the steel company and the iron corporation in federal court to enjoin the merger, alleging that it was in violation of the federal antitrust laws. The federal district judge enjoined the merger, and the steel company appealed the judge's decision. The steel company's lawyer, doing the legal research for the appeal, found a recent merger decision rendered by the Federal Trade Commission ("FTC") that is directly adverse to the steel company's position. FTC decisions do not control in the United States Courts of Appeal, but they are persuasive. The attorney general failed to cite the FTC decision. Must the steel company's attorney disclose it to the court? response - incorrect A Yes, because it is persuasive authority. B Yes, because the FTC decision is directly adverse to the steel company's position. C No, because an attorney has no obligation to volunteer facts harmful to his client's case. D No, because the court of appeals is not obliged to follow the FTC ruling.

D. The steel company's lawyer need not disclose the FTC ruling because the court need not follow the decision. An attorney can be disciplined for failing to cite the court to legal authority that is "directly adverse" to the client's position and is from the "controlling jurisdiction." [ABA Model Rule 3.3(a)(2)] Although the decision is directly adverse to the steel company's position, the facts state that FTC decisions do not control in the United States Courts of Appeal. Thus, the decision is not from the "controlling jurisdiction." (A) is wrong because in order to invoke the disclosure rule, the decision must be from a controlling jurisdiction, not merely persuasive. (B) is wrong because it states only one portion of the test that triggers the disclosure rule. While true that the decision is adverse, it is not from a controlling jurisdiction, and thus need not be revealed. (C) is wrong because although the Rule as stated is true, in this case we are not concerned with harmful facts. The issue is whether the steel company's attorney must reveal harmful law.

A prospective client comes to a law office seeking a lawyer to defend him in a civil action for aggravated assault and battery. A lawyer agrees to talk preliminarily with the client, just to obtain enough background information to decide whether she can defend him. The client explains that he has an alcohol problem; indeed, he gets roaring drunk about three nights a week. On the night in question, the client said that a loud-mouthed stranger in his neighborhood tavern made a derogatory comment about the client's favorite basketball team. The client responded by "tapping" the stranger over the head with a pool cue, not once but four times. At that point, the lawyer suddenly realizes that the client must be the rotten husband in the hotly disputed divorce and child custody case in which her law partner is representing the aggrieved wife. The lawyer stops the client and tells him that she cannot defend him in the assault and battery case because of her partner's work for the client's wife. Which of the following is true? A The partner must withdraw from representing the wife because the lawyer has received confidential information from the client that would be harmful to the client if used in the divorce and child custody case. B It would be proper for the partner to represent the wife and for the lawyer to represent the client in the assault and battery case because the two matters are not substantially related. C The partner may continue representing the wife, but only if the wife gives informed consent, confirmed in writing. D The partner may continue representing the wife if the lawyer is screened off from participation in the case and obtains no part of the fee in the case, and if the firm promptly sends the client written notice of the situation.

D. This question is governed by ABA Model Rule 1.18, which concerns duties to a prospective client. The information that the lawyer obtained about the prospective client's alcohol abuse and his violent response to the stranger's comment could be harmful to the prospective client if the wife uses it to help prove that the couple should be divorced and that the prospective client should not be given custody of their children. Therefore, the lawyer herself could not represent the wife in the divorce and child custody case. [See ABA Model Rule 1.18(c)] The lawyer's disqualification is imputed to her law partner. [Id.] However, if the conditions mentioned in (D) are satisfied, then her law partner may continue representing the wife. [See ABA Model Rule 1.18(d)(2)] (C) is wrong because it calls for informed consent by the wife only.

A private adoption agency handles over 65% of all private adoptions in the state in which the agency is located. The agency provides each set of prospective adoptive parents with a list of lawyers whom the agency recommends and tells them that it is in their best interest to obtain a lawyer experienced in adoptions. The agency has investigated each lawyer on its list to ensure the lawyer's experience and reputation for honesty and ethical behavior. To further protect prospective adoptive parents, the agency also requires each lawyer on the list to agree not to withdraw from any case where he has been retained by parents adopting through the agency. A lawyer who has handled many private adoptions and is highly regarded as an honest and competent lawyer would like to be placed on the agency's list. Would it be proper for the lawyer to sign the agreement and have himself included in the list? response - incorrect A Yes, because the list is a valuable service, and people often do not know where to find a lawyer qualified to handle a matter such as a private adoption. B Yes, because the lawyer does not give anything of value to the agency in exchange for being placed on the list. C No, because the agency is acting as the lawyer's agent, and a lawyer may not use an agent to contact persons in a manner that would be unethical solicitation if done by the lawyer. D No, because the agreement allows a third party to exercise influence over the lawyer-client relationship.

D. It would not be proper for the lawyer to have his name included on the agency's list because the agreement allows a third party to exercise influence over the lawyer-client relationship. A lawyer must not allow a person who recommends, employs, or pays her for serving a client to direct or regulate the lawyer's professional judgment. [ABA Model Rule 5.4(c)] The agency is, in effect, recommending the lawyer, and the restriction on withdrawal in the form agreement clearly interferes with the lawyer's professional judgment. (A) is wrong because the good intentions behind the list do not remove the interference with the lawyer's professional judgment. Likewise, (B) is wrong because, even if the lawyer does not give the agency anything of value, the arrangement is still improper because of the restriction on withdrawal. (C) is wrong because the list does not amount to improper solicitation.

For the past five years, an attorney has represented an art dealer in the sale of many valuable paintings. One of the major transactions occurred three years ago, when the art dealer sold a landscape purportedly painted by Vincent van Gogh to an art museum for $23 million. The museum subsequently resold the painting for $35 million. Now the art dealer has asked the attorney to do the legal work in connection with the sale of another landscape, also a purported van Gogh. The proposed purchase price is $12 million, and the prospective purchaser is a wealthy television personality who knows nothing about art. During a confidential conversation in the attorney's office, the attorney asked the art dealer if he had appraisal letters certifying the painting as a genuine van Gogh. The art dealer replied that he indeed had letters—letters he had forged himself—and that he had also forged the letters for the purported van Gogh sold to the art museum. When the attorney inquired further, the art dealer told him in confidence that both of the purported van Gogh paintings were in fact counterfeits created by a clever art student. Which of the following must the attorney do at this point? A Report the art dealer to the law enforcement authorities. B Warn the prospective purchaser about the proposed sale. C Inform the art museum of the truth about the first painting. D Refuse to represent the art dealer in the present transaction.

D. The attorney must refuse to represent the art dealer in the present transaction. Note that the call of the question asks what the attorney must do, not what he would be allowed to do. ABA Model Rule 1.2 states that a lawyer must not counsel or assist a client in conduct that the lawyer knows is criminal or fraudulent. Here, the attorney must refuse to represent the art dealer in the sale of the painting to the television personality because the sale would be fraudulent—the attorney knows that the painting is not a van Gogh as the art dealer represented to the television personality. ABA Model Rule 1.6(b) permits a lawyer to reveal a client's confidential information to the extent necessary to prevent the client from committing a crime or fraud that would result in substantial financial harm to a person, if the client is using or has used the lawyer's services to further the crime or fraud. A lawyer also may reveal confidential information if the client has already acted and the disclosure will mitigate the consequent financial harm. Thus, the attorney is permitted to disclose information to law enforcement authorities in order to prevent harm to the television personality and possibly mitigate subsequent financial harm caused by the first transaction to the art museum; however, the attorney is not required to do so. The same holds true regarding the attorney's warning the television personality about the proposed sale. Consequently, (A) and (B) are wrong. (C) is also wrong because even if it can be argued that the attorney's revealing information about the fraudulent sale to the museum would mitigate subsequent financial harm, the attorney is not required to reveal the art dealer's confidential information.

An elderly widower has one living child, a daughter. The widower's main asset is a 51% partnership interest in a wealthy real estate syndicate that owns and operates mobile home parks throughout the state. The daughter's husband is an attorney. One of the husband's regular clients asks the husband to represent him in negotiating the sale of 3,000 acres of roadside property to the real estate syndicate. The real estate syndicate is represented by its own lawyer in the matter. May the husband represent his regular client in a sale with the real estate syndicate? response - incorrect A No, even if the client gives informed consent, confirmed in writing. B No, because to do so would create an appearance of impropriety. C Yes, because the husband has no significant personal interest in the real estate syndicate. D Yes, but only if the client gives informed consent, confirmed in writing.

D. Conflict of Interest- need informed consent in writing The husband may represent his regular client if the client gives informed consent, confirmed in writing, to the representation. His wife is likely to inherit her father's interest in the real estate syndicate. That gives the husband a personal interest in the real estate syndicate, albeit an attenuated interest. If the husband is to represent his regular client in selling land to the real estate syndicate, he must first disclose his personal interest to the client. If the client gives informed consent, confirmed in writing, then the husband may represent the client. [ABA Model Rule 1.7(b)] (A) is wrong because informed, written consent will solve the potential conflict of interest. (B) is wrong because informed consent, confirmed in writing, will solve the conflict problem. Furthermore, the "appearance of impropriety" is not a basis for discipline or disqualification under the ABA Model Rules. This is an outdated concept from the old ABA Model Code. (C) is wrong because the husband's personal interest is significant, even though it is remote.

An amateur inventor comes to an attorney with an invention he wants to patent. The inventor explains that he and a competitor have been racing one another to come up with the ideal cleaning solution. If the inventor's competitor were to find out that the inventor was at the patent stage, and worse, if he found out the inventor's formula, the inventor would be ruined. The attorney, a trained and certified patent attorney, agrees to represent the inventor in the patent process. The invention involves complex chemical formulae, and the attorney's particular area of expertise is electronic devices. However, having worked with inventions of all types, she has no doubt that she can properly shepherd the solution through the patent process. In putting together the necessary paperwork, the attorney asks two associates in her firm who hold chemistry degrees to help her out on the project. In due time, the inventor's product receives a patent. The total bill for legal fees was $60,000, which was reasonable for the work done. When the attorney received the inventor's final payment, she decided to give the two associates each a $10,000 bonus from the fee. Are the attorney's actions proper? response - incorrect A Yes, because with the aid of her associates, she was competent to handle the matter. B Yes, because the division of the fee was in proportion to the work performed by the two associates. C No, because the attorney shared information about the case with the two associates. D No, because the inventor did not consent to the splitting of the fee with the two associates.

D. Same firm so fee splitting is okay. The attorney's actions are proper. With the aid of her associates, the attorney, an experienced patent lawyer, was clearly competent to handle the case. (B) is wrong because it states one of the limitations on fee splitting between lawyers in different firms. It does not apply to lawyers in the same firm. (C) is wrong because this is the type of disclosure that is impliedly authorized to carry out the representation. It does not abrogate the protections of privilege or confidentiality, and no formal consent by the client is required. [See ABA Model Rule 1.6(a)] (D) is wrong because it too states a limitation concerning fee splits between lawyers in different firms. No client consent is required to split a fee in any manner among lawyers in the same firm. [ABA Model Rule 1.5]

A consumer who bought a defective product that injured him hired a lawyer to represent him in a personal injury action against the large corporation that made the product. As the consumer and the lawyer discussed the case, the consumer stated that he probably would not agree to a settlement under $500,000. The lawyer agreed that the claim was worth at least that, but felt they would receive a much higher award if the case went to a jury. Shortly before the trial started, the lawyer for the corporation contacted the consumer's lawyer with a settlement offer of $150,000. The consumer's lawyer tried to call his client, but could not reach him. After two hours of trying to reach his client, the attorney called opposing counsel and rejected the offer. At trial, the jury awarded the consumer $1 million. Is the consumer's lawyer subject to malpractice liability for his actions? response - incorrect A Yes, because a lawyer has a duty to keep his client informed of all settlement offers. B Yes, because decisions to accept or reject settlement offers are to be made by the client. C No, because the consumer impliedly authorized his lawyer to reject any offer under $500,000. D No, because the jury award was much greater than the settlement offer.

MALPRACTICE; not discipline, trick question D. The consumer's lawyer is not subject to malpractice liability because his client (the consumer) has no damages. Damages are part of the cause of action for legal malpractice. The lawyer may be subject to discipline, but not malpractice liability. (A) is incorrect because, although it correctly states the lawyer's duty, there is no malpractice claim because of the lack of damages. (B) is incorrect for the same reason. (C) is incorrect because the consumer's statement that he probably would not accept an offer under $500,000 does not constitute authorization of the lawyer to accept or reject offers without consulting him. Furthermore, to avoid problems such as this, authorization to accept or reject settlement offers without consulting the client should be in writing.

When a lawyer was an associate at a law firm, she did the legal work for one of the firm's clients on a land sale transaction that earned the client millions of dollars. In gratitude, the client asked the lawyer whether she had any unfulfilled wishes. The lawyer told the client that she wished she had enough money to start her own solo law practice. The client then told her that he would lend her $100,000 to set up her new practice. In return, she would thereafter do all of his legal work at a 5% discount from her normal hourly fee, and she would pay off the $100,000 loan by monthly payments equal to 10% of her net income for the prior month. The lawyer was delighted. She drafted a complete, detailed agreement between herself and the client, and she advised the client in writing to obtain outside legal advice before signing the agreement. The client obtained the outside advice and signed the agreement, and the lawyer set up her solo practice accordingly. Is the lawyer subject to discipline? response - incorrect A No, unless she allows the client to interfere with her professional judgment in handling work for other clients. B No, unless the lawyer fails to give the law firm timely notice of the transaction. C Yes, unless the law firm consented to the loss of the client as a firm client. D Yes, unless the client is a lawyer.

The lawyer is not subject to discipline unless she allows the client to interfere with her judgment in handling other clients' matters. [See ABA Model Rule 5.4] (B) is wrong because there is no law or disciplinary rule that requires the lawyer to notify the law firm regarding the transaction. (C) is wrong because no law or disciplinary rule requires the law firm's consent to the client's leaving the firm and giving his business to the lawyer as a solo practitioner. (D) is wrong because the loan payback clause does not violate the rule against splitting a legal fee with a nonlawyer. [ABA Model Rule 5.4(a)] True, the clause does measure the monthly payments as a percentage of the lawyer's net income in the prior month and most of her net income will probably come from legal fees. However, it makes sense to tailor her loan payments to her income, and the arrangement does not invite the evil that the no-splitting rule was designed to prevent—interference with the lawyer's professional judgment.


Set pelajaran terkait

English 2 - Test: Passage Response -

View Set

10-Classroom Language- what the learner says

View Set

Chapter 27 study questions (A/P2)

View Set

Econ Chap. 3 Section 1 Guided Reading

View Set

Partial Income Statement Preparation

View Set